Categories
Chicago Economics Programs

Chicago. Program of Political Economy. Thick course descriptions. 1904-1905

Broschures that advertise economics departments are often useful summaries of the “order of battle” for their educational and research missions. The Chicago Department of Political Economy was about a dozen years in business when this programme, transcribed below, was published. The course descriptions are somewhat thicker than are typically found in full university catalogs that must share space for the many divisions and schools that constitute the larger institution. 

Incidentally, the copy of the printed programme that was transcribed for this post was found in an archival box of material dealing with graduate studies in the Division of History, Government, and Economics at Harvard University. Then as now, prudence demands keeping an eye on your competition. 

______________________________

Related Posts on the Early Years
of the Department of Political Economy
of the University of Chicago

First detailed announcement of Political Economy program at the University of Chicago, 1892.

General Regulations for the degree of Ph.D. at the University of Chicago, 1903.

Twenty-Fifth Anniversary of the Department of Political Economy, 1916.

______________________________

CONSPECTUS OF COURSES,
1904-1905.
POLITICAL ECONOMY.

All courses are Mj [major] unless otherwise indicated.

SUMMER AUTUMN WINTER SPRING
1 Principles of Political
  Economy
(Hill) 9:00
1 Principles of Political Economy
a (Hill) 8:30
b (Davenport) 12:00
1 Principles of Political Economy
(Hill) 12:00
2 Principles of Political Economy Con’d
a (Hill) 8:30
b (Davenport) 12:00
2 Principles of Political Economy Con’d
(Davenport) 12:00
3 Economic and Social History
(Morris) 2:00
3 Economic and Social History
(Morris) 12:00
4 History of Commerce
(Morris) 12:00
5B Commercial Geography for Teachers
(Goode) 1:30
5 Commercial Geography
(Mr. —) 8:30
8 Mathematical Problems of Insurance
(Epsteen)
6 Modern Industries
(Mr. —) 11:00
7 Insurance
(Davenport) 8:30
[9 Law of Insurance]
(Bigelow)
10 Accounting
(Mr. — ) 11:00
11 Special Problems of Accounting
(Several Experts)
12 Modern Business Methods
(Clow) 8:00
12 Modern Business Methods
(Mr. —) 9:30
20 History of Political Economy
(Veblen) 11:00
21 Scope and Method
(Veblen) 11:00
22 Finance
(Davenport) 8:30
24 Financial History of the United States
(Cummings) 8:00
24 Financial History of the United States
(Cummings) 2:00
23 Tariff Reciprocity and Shipping
(Cummings) 9:30
26 American Agriculture
(Hill) 10:30
26 American Agriculture
(Hill) 10:30
25 Economic Factors in Civilization
(Veblen) 11:00
27 Colonial Economics
(Morris) 9:30
40 Value
(Davenport) 8:30
41 Labor and Capital
(Laughlin) 12:00
44 Socialism
(Veblen) 9:30
46 Trade Unions
(Cummings)
9:30
45 Industrial Combinations (Veblen) 9:30 43 Economics of Workingmen
(Cummings) 9:30
46 Trades Unions
(Cummings) 12:00
50 Money
(Laughlin) 12:00
51 Banking
(Mr. —) 8:30
50 Money
(Laughlin) 12:00
[52 Advertising]

53 Practical Banking
(Mr. — ) 8:30

60 Railways
(Hill) 2:00
61 Railway Rates
(Meyer) 2:00
62 Government Ownership (I)
(Meyer) 2:00
63 Government Ownership (II)
(Meyer) 2:00
64 American Competition
(Meyer) 3:00
70 Statistics
(Cummings) 8:30
71 Statistics of Wages
(Cummings) 12:00
80 Seminar
(Laughlin)
81 Seminar
(Laughlin)
[82 Seminar
(Laughlin)]

______________________________

THE DEPARTMENT
OF POLITICAL ECONOMY.

OFFICERS OF INSTRUCTION.

JAMES LAURENCE LAUGHLIN, Ph.D., Professor and Head of the Department of Political Economy.

THORSTEIN B. VEBLEN, Ph.D., Assistant Professor of Political Economy.

WILLIAM HILL, A.M., Assistant Professor of Political Economy.

JOHN CUMMINGS, Ph.D., Assistant Professor of Political Economy.

HENRY RAND HATFIELD, Ph.D., Assistant Professor of Political Economy.

HERBERT JOSEPH DAVENPORT, Ph.D., Assistant Professor of Political Economy.

HUGO RICHARD MEYER, A.B., Assistant Professor of Political Economy.

ROBERT MORRIS, A.B, LL.B., Instructor in Political Economy.

F. R. CLOW, Professor of Political Economy, State Normal School, Oshkosh, Wis. (Summer Quarter, 1904).

FELLOWS.
1904-1905.

EDITH ABBOTT, A.B
EARL DEAN HOWARD, Ph.B.
WILLIAM JETT LAUCK, A.B.

INTRODUCTORY.

The work of the department is intended to provide, by symmetrically arranged courses of instruction, a complete training in the various branches of economics, beginning with elementary work and passing by degrees to the higher work of investigation. A chief aim of the instruction will be to teach methods of work, to foster a judicial spirit, and to cultivate an attitude of scholarly independence. (1) The student may pass, in the various courses of instruction, over the whole field of economics. (2) When fitted, he will be urged to pursue some special investigation. (3) For the encouragement of research and the training of properly qualified teachers of economics, Fellowships in Political Economy have been founded. (4) To provide a means of communication between investigators and the public, a review, entitled the Journal of Political Economy, has been established, to be edited by the officers of instruction in the department; while (5) larger single productions will appear in a series of bound volumes to be known as Economic Studies of the University of Chicago. [For links see below]

FELLOWSHIPS.

The Fellowships here offered by the Department of Political Economy are independent of those offered by the allied departments of History, Political Science, or Sociology.

Appointments will be made only on the basis of marked ability in economic studies and of capacity for investigation of a high character. It is a distinct advantage to candidates to have been one year in residence at the University. Candidates for these Fellowships should send to the President of the University a record of their previous work and distinctions, degrees and past courses of study, with copies of their written or printed work in economics. Applications should be sent in not later than March 1 of each year Appointments will be made during the first week of April.

Fellows are forbidden to give private tuition, and may be called upon for assistance in the work of teaching in the University or for other work; but in no case will they be expected or permitted to devote more than one sixth of their time to such service.

In addition, one Graduate Scholarship, yielding a sum sufficient to cover the annual tuition fees, is awarded to the best student in economics just graduated from the Senior Colleges; and a similar Scholarship is given to the student graduating from the Junior Colleges who passes the best examination at a special test.

CANDIDACY FOR HIGHER DEGREES.

Graduate courses are provided for training and research in subjects such as wages, money, agriculture, socialism, industrial combinations, statistics, demography, finance, and the like. Specialization may be carried on in many parts of the field, under special direction in the Seminar, whereby each student receives a personal appointment for one hour a week. The work is so adjusted as to form an organized scheme leading by regular stages to productive results suitable for publication.

Candidates for the degree of A.M. will not be permitted to offer elementary courses in Political Economy as part of the work during the year’s residence. The work of students taking Political Economy as a secondary subject for the degree of A.M., should include (1) the general principles of economics (as contained in Courses 1 and 2, or an equivalent); (2) the history of Political Economy; and (3) the scope and method of Political Economy.

The work of candidates for the degree of Ph.D., taking Political Economy as a secondary subject, should include, in addition to the above requirements for the degree of A.M., on (1) Public Finance, and (2) on some descriptive subject as, e.g., Money, or Tariff, or Railways, etc.; and the examination will be more searching than that for the degree of A.M.

In all cases candidates should consult early with the heads of the departments within which their Major and Minor subjects are taken.

Before being admitted to candidacy for the degree of Doctor of Philosophy, in case Political Economy is chosen as the principal subject, the student must furnish satisfactory evidence to the head of the department that he has been well prepared in the following courses (or their equivalents at other institutions): History of Europe in the Seventeenth and Eighteenth Centuries (History 11); Europe in the Nineteenth Century (History 12); Later Constitutional Period of the United States; the Civil War and the Reconstruction (History 18); Comparative National Government (Political Science 11); Federal Constitutional Law of the United States (Political Science 21); Elements of International Law (Political Science 41); and Introduction to Sociology (Sociology 72).

PUBLICATIONS

As a means of communication between investigators and the public, the University issues quarterly the Journal of Political Economy, the first number of which appeared in December of 1892. Contributions to its pages will be welcomed from writers outside as well as inside the University the aim being not only to give investigators a place of record for their researches, but also to further in every possible way the interests of economic study throughout the country. The Journal will aim to lay more stress than existing journals upon articles dealing with practical economic questions. The editors will welcome articles from writers of all shades of economic opinion, reserving only the privilege of deciding as to merit and timeliness.

Longer investigations, translations of important books needed for American students, reprints of scarce works, and collections of materials will appear in bound volumes in a series of Economic Studies of the University of Chicago, of which the following have already been issued:

No. I. The Science of Finance, by Gustav Cohn. Translated by Dr. T. B. Veblen, 1895, 8vo, pp. xi+800. Price, $3.50.

No. II. History of the Union Pacific Railway, by Henry Kirke White, 1895, 8vo, pp. 132. Price, $1.50.

No. III. The Indian Silver Currency, by Karl Ellstaetter. Translated by J. Laurence Laughlin, 1896, 8vo, pp. 116. Price, $1.25.

No. IV. State Aid to Railways in Missouri, by John Wilson Million, 1897, 8vo, pp. 264. Price, $1.75.

No. V. History of the Latin Monetary Union, by Henry Parker Willis, 1901, 8vo, pp. ix + 332. Price $2.00.

No. VI. The History of the Greenbacks, with Special Reference to the Economic Consequences of Their Issue, by Wesley Clair Mitchell, 1903, 8vo, pp. xiv + 500. Price, $4.00, net.

No. VII. Legal Tender: A Study in English and American Monetary History, by Sophonisba P. Breckinridge, 1903, 8vo, pp. xvii + 180. Price, $1.50, net.

LIBRARY FACILITIES.

In the suite of class-rooms occupied by the department will be found the Economic Library. Its selection has been made with great care, in order to furnish not only the books needed for the work of instruction in the various courses, but especially collections of materials for the study of economic problems. The University Library contains an unusually complete set of United States Documents, beginning with the First Congress. It is believed that ample provision has thus been made for the work of serious research. The work of the students will necessarily be largely carried on in the Economic Library where will also be found the past as well as the current numbers of all the European and American economic journals.

The combined library facilities of Chicago are exceptional. The Public Library, maintained by a large city tax, the Newberry Library, and the Crerar Library, with a fund of several millions of dollars, which has provided books on Political Economy, will enable the student to obtain material needed in the prosecution of detailed investigation.

SPECIAL ADVANTAGES.

For the convenience of those who wish to know the branches of economics in which especial advantages are offered by the department, attention is called to the new facilities afforded for specialization in several directions:

RAILWAYS.

Apart from the fundamental training in the general economic field, a new and exceptional series of advanced courses in the economic side of railways has been provided. It is believed that no such extended and useful courses have ever been offered before on this subject. Beginning with the usual general course on railway transportation, four new courses are presented for advanced students.

LABOR AND CAPITAL.

In view of the pressing importance of questions touching upon the rewards of labor and capital, an exceptional arrangement of courses dealing both with the underlying principles and the practical movements of the day have been prepared upon new and extended lines.

MONEY.

Opportunities for specialization in the field of money and banking have been offered in the past, but new courses have been organized in order to permit a more thorough study in these subjects, both theoretical and practical, than has ever been possible before.

LABORATORY FOR STATISTICAL RESEARCH WORK.

The University has equipped a laboratory for statistical research work in which students are given training in the collection and tabulation of statistical data, as well as in the scientific construction of charts, and diagrams. The object of the work is to familiarize students with practical methods employed in government bureaus, municipal, state, and federal, in the United States and in other countries, and in private agencies of sociological and economic investigation. Men are trained to enter the service of such bureaus or agencies of social betterment as statisticians, capable of undertaking any work requiring expert statistical service. The Departments of Political Economy and of Sociology co-operate in the direction of statistical investigations.

COURSES OF INSTRUCTION.
Summer Quarter, 1904—Spring Quarter, 1905.

M=Minor course a single course for six weeks.
Mj=Major course=a single course for twelve weeks

GENERAL.

The courses are classified as follows:

Group 1, Introductory and Commercial: Courses 1, 2, 3, 4, 5, 6.

Group II, Advanced Business Courses: Courses 7, 8, 9, 10, 11, 12.

Group III, General Economic Field: Courses 20, 21, 22, 23, 24, 25, 26, 27, 29-30, 31-32.

Group IV, Labor and Capital: Courses 40, 41, 42, 43, 44, 45, 46, 47.

Group V, Money and Banking: Courses 50, 51, 52, 53, 54.

Group VI, Railways: Courses 60, 61, 62, 63, 64.

Group VII, Statistics: Courses 70, 71, 72.

Group VIII, Seminars: Courses 80, 81, 82.

Students are advised to begin the study of economics not later than the first year of their entrance into the Senior Colleges; and students of high standing, showing special aptitude for economic study, may properly take the Courses of Group I in the last year of the Junior Colleges.

For admission to the courses of Groups II and III, a prerequisite is the satisfactory completion of Courses 1 and 2 in the department, or an equivalent. Course 1 is not open to students who do not intend to continue the work of Course 2.

JUNIOR COLLEGE COURSES.
Group I. — Introductory and Commercial. 

1 and 2. Principles of Political Economy. — Exposition of the laws of modern Political Economy.

Course 1.

Mj. Summer Quarter; 8:00. Assistant Professor Hill.

Mj. Autumn Quarter; 2 sections, 8:30 and 12:00. Assistant Professors Hill and Davenport.

Mj. Winter Quarter; 12:00. Assistant Professor Hill.

Course 2.

Mj. Winter Quarter; 2 sections: 8:30 and 12:00. Assistant Professors Hill and Davenport.

Mj. Spring Quarter; 12:00. Assistant Professor Davenport.

Courses 1 and 2 together are designed to give the students an acquaintance with the working principles of modern Political Economy. The general drill in the principles cannot be completed in one quarter; and the department does not wish students to elect Course 1 who do not intend to continue the work in Course 2. Descriptive and practical subjects are introduced as the principles are discussed, and the field is only half covered in Course 1. Those who do not take both 1 and 2 are not prepared to take any advanced courses.

  1. Economic and Social History. — It is thought that the course may be of advantage to students of Political Science and History by giving them a view of the economic side of the social and political life of the past one hundred and fifty years. Special attention is devoted to the study of the economic effects of the Colonial System; the American and French Revolutions; the “industrial revolution;” the effects of invention and the new transportation upon the movement and grouping of population; the discoveries of the precious metals in North America, South America, Africa, and Australia; slavery, the Civil War, the new South, and the redistribution of industries in the United States; the progress of Great Britain since the repeal of the Corn Laws; and the recent development of German industry.

Mj. Autumn 2:00 and Spring Quarters; 12:00.
Mr. Morris.

Course 3 is required of all students in the College of Commerce and Administration.

  1. History of Commerce. — A brief general survey of ancient medieval and modern commerce. Consideration of the articles of commerce, the market places, the trade routes, methods of transportation, and the causes which promoted and retarded the growth of commerce in the principal commercial nations.

Mj. Autumn Quarter; 12:00. Mr. Morris.

  1. Commercial Geography. — A study of the various countries and their chief products; the effect of soil, climate, and geographical situation in determining national industries and international trade, commercial routes, seaports; the location of commercial and industrial centers; exports and imports; the character, importance, and chief sources of the principal articles of foreign trade.

Mj. Autumn Quarter; 8:30.
Mr. ———

Required of all students in the
College of Commerce and Administration.

  1. Modern Industries. — This elementary course, requiring no previous study of economics, examines the present organization of some of the leading industries. Study is made of the internal business organization, the processes of manufacture, the effect of inventions, etc. Emphasis will be placed on the manufactures of the United States.
    The class will visit a number of large industrial establishments in and near Chicago.

Mj. Autumn Quarter; 11:00.
Mr. ———

SENIOR COLLEGES AND GRADUATE COURSES.
Group II. — Advanced Business Courses.
  1. Insurance. — This course will aim to cover those aspects of insurance important to the practical business man, and to serve at the same time as a descriptive and theoretical treatment adapted to the needs of students intending to specialize in the actuarial and legal aspect of the subject. The history and theory of insurance, the bearing of these on the different insurance relations of modern business, including accident, health, burial, suretyship, credit forms, and the like will be examined. Especial emphasis will, however, be given: (1) to Life Insurance, the various forms of organization, assessment, fraternal, stock, and mutual; the theory of rates, mortality, expense, reserve, and interest aspects; the different combinations of investment and mortality contracts, loan and surrender values, dividends, distribution periods; (2) to Fire Insurance, the various forms of business organization, the terms and conditions of the insurance contract, the different forms of hazard, and the competition and combination of rates therefor; the theory of reserves and co-insurance, and the problem of the valued policy laws; (3) the general principle of public supervision with regard to the different forms of insurance, and the wider question of public ownership.

Mj. Winter Quarter; 8:30.
Assistant Professor Davenport.

  1. The Mathematics of Insurance. — This course presupposes some acquaintance with the descriptive aspect of insurance. The course is devoted particularly to the mathematical principles of Life Insurance. The necessary elements of the theory are selected from the theories of probability, finite differences, and interpolation. Applications are made in particular to the following problems: The examination of the different mortality tables and the basing of mortality rates thereon; the loading of expenses and reserves and the variations of premiums, as affected by the prospective earnings of investments: the computation of total reserves; the fixation of loan and surrender values of paid-up insurance, whether by life or term extension; the computation of present and deferred annuities as affected by considerations of age, life, term, endowment, joint-life, and annuity policies.

Mj. Spring Quarter; 12:00.
Mr. Epsteen.

Prerequisite: Trigonometry and College Algebra
(Mathematical Courses 1, 2 or 1, 5 or 4, 5).
See Mathematics 9.

  1. Law of Insurance. — Insurable interest in various kinds of insurance and when it must exist; beneficiaries; the amounts recoverable and valued policies; representations; warranties; waiver and powers of agents; interpretation of phrases in policies; assignment of insurance.

Mj. Spring Quarter.
Assistant Professor Bigelow.

Text book: Wambaugh, Cases on Insurance.

  1. Accounting. — The interpretation of accounts viewed with regard to the needs of the business manager rather than those of the accountant: the formation and meaning of the balance sheet; the profit and loss statement and its relation to the balance sheet; the capital accounts, surplus, reserve, sinking funds; reserve funds, their use and misuse; depreciation accounts; other accounts appearing on credit side; assets; methods of valuation; confusing of assets and expenses; capital expenditures and operating expenses; capital assets, cash, and other reserves.

Prerequisite: The Course in Bookkeeping offered by the Department of Mathematics.

Mj. Winter Quarter; 11:00.
Mr. ———.

  1. Special Problems in Accounting.
    1. Bank accounting.
    2. The duties of an auditor; methods of procedure; practice; problems frequently met.
    3. Appraisal and Depreciation.
    4. Railway Accounting. A consideration of the principal features. Determination of the four main divisions of expense. The relation between capital expenditures and profit and loss.
    5. The Public Accountant. Legal regulations; duties and methods; constructive work in devising system of accounting to fit special needs. Practice in comparison of various systems. The advantages of various devices, loose-leaf and card systems; voucher system; cost keeping.

Mj. Spring Quarter.
Conducted by experts from Chicago institutions.

  1. Modern Business Methods. Corporation Finance. — Speculation, investment, exchange. The course aims to make clear to the student the meaning of the commercial and financial columns of current journals and to examine the economic significance of the business transactions thus reported. Attention is given among other things to the reports of the money market, the business on stock and produce exchanges, market quotations, the various forms of investment securities, and foreign exchange.

Summer Quarter; 11:30.
Mj. Spring Quarter; 9:30.
Professor Clow.

Group III — General Economic Field.
  1. History of Political Economy. — Lectures, Reading, and Reports. This course treats of the development of Political Economy as a systematic body of doctrine; of the formation of economic conceptions and principles, policies, and systems. The subject will be so treated as to show the continuity and systematic character of Political Economy as an intelligent explanation of economic facts. Both the history of topics and doctrines and that of schools and leading writers will be studied. Attention will be given to the commercial theories of the Mercantile System, the Physiocratic School, Adam Smith and his immediate predecessors, the English writers from Adam Smith to J. S. Mill, and the European and American writers of the nineteenth century. Selection will be made of those who have had great influence, and who have made marked contributions to Political Economy. The student will be expected to read prescribed portions of the great authors bearing on cardinal principles. It is hoped that in this way he will learn to see the consistency and relations of economic theories and to use the science as a whole, and not as a mere mass of arbitrary formulæ or dicta. A special feature of the work will be a thorough study of Adam Smith and of Ricardo.

Mj. Autumn Quarter; 11:00.
Assistant Professor Veblen.

  1. Scope and Method of Political Economy. — The course treats of the premises on which the analysis of economic problems proceeds, the range of problems usually taken up for investigation by economists, the methods of procedure adopted in their solution, the character of the solutions sought or arrived at, the relations of Political Economy to the other Moral Sciences, as well as to the influence of the political, social, and industrial situation in determining the scope and aim of economic investigation. Special attention is given to writers on method, as Mill, Cairnes, Keynes, Roscher, Schmoller, Menger.

Mj. Winter Quarter; 11:00.
Assistant Professor Veblen.

  1. Finance. — In this course it is intended to make a comprehensive survey of the whole field of public finance. The treatment is both theoretical and practical, and the method of presentation historical as well as systematic. Most emphasis is placed upon the study of taxation, although public expenditures, public debts, and financial administration are carefully studied.

Mj. Spring Quarter; 8:30.
Assistant Professor Davenport.

  1. Tariffs, Reciprocity, and Shipping. — The course of legislation and the development of our commercial policy is followed, and an effort made to indicate the influence of our protective tariffs upon the development of our domestic industries, upon the growth and character of our international trade, and incidentally upon the occurrence of industrial crises and the continuance of industrial prosperity at different periods in our history. Foreign trade policies and schemes for imperial tariff federation are taken up, and especial attention given to the negotiation of reciprocity treaties, as well as to recent attempts which have been made through federal legislation granting subsidies to build up American shipping.

Mj. Spring Quarter; 11:00.
Assistant Professor Cummings.

  1. Financial History of the United States. — In this course the financial history of the United States is followed from the organization of our national system in 1789 to the close of the Spanish war. The following topics may be mentioned as indicating the scope of the course; the funding and management of the Revolutionary and other war debts; the First and Second United States Banks; the Independent Treasury; the present national banking system; Civil War financiering with especial reference to bond and note issues, and resort to legal tender currency; the demonetization of silver and issue of silver certificates; inflation of the currency and the gold reserve; the currency act of 1900. This study of the course of legislation upon currency, debts, and banking in the United States is based upon first-hand examination of sources, and students are expected to do original research work.

Summer Quarter; 8:00.
Mj. Autumn Quarter; 9:30.
Assistant Professor Cummings.

  1. Economic Factors in Civilization. — The course is intended to present a genetic account of the modern economic system by a study of its beginnings and the phases of development through which the present situation has been reached. To this end it undertakes a survey of the growth of culture as affected by economic motives and conditions. With this in view, such phenomena as the Teutonic invasion of Europe, the Feudal system, the rise of commerce, the organization of trade and industry, the history of the condition of laborers, processes of production, and changes in consumption, will be treated.

Mj. Spring Quarter; 11:00.
Assistant Professor Veblen.

  1. Problems of American Agriculture. — Special attention will be given to the extension and changes of the cultivated area of the United States; the methods of farming; the influence of railways and population, and of cheapened transportation; the fall in value of Eastern farm lands; movements of prices of agricultural products; European markets; competition of other countries; intensive farming; diminishing returns; farm mortgages; and the comparison of American with European systems of culture.

Summer Quarter; 10:30.
Mj. Winter Quarter; 9:30.
Assistant Professor Hill.

  1. Colonial Economics. — The economics of colonial administration, including some account of commercialism, past and present, and of modern trade theories of imperial federation, trade relations, financial policies, and economic development and dependence of colonies.
    A brief historical account of American and foreign experience serves as introduction to a fuller consideration of economic problems involved in modern colonial administration. In the light of this experience study is undertaken of some economic problems which have arisen in Cuba, Porto Rico, Hawaii, and the Philippine Islands.

Mj. Spring Quarter; 9:30.
Mr. Morris.

29, 30. Oral Debates. — Selected Economic Topics. Briefs. Debates. Criticism.

2M. Autumn and Winter Quarters; Mon., 3:00-6:00.
Assistant Professor Hill, Mr. Chandler, and Mr. Gorsuch.

31, 32. Argumentation. — To be taken in connection with English 9.

2 hrs. a week.
2M. Autumn and Winter Quarters; Wed., 3:00.
2M. Autumn and Winter Quarters.
Mr. Chandler.

Group IV. — Labor and Capital.
  1. Theory of Value. — After a brief preliminary survey of the discussions prior to Adam Smith, the cost of production-theory as developed at the hands of Ricardo, McCulloch, James Mill, Senior, J. S. Mill, and Cairnes is taken up for detailed study. Then the utility theory of value, as presented by Jevons and Austrian economists, is examined. Finally, the attempts made by such writers as Marshall, Dietzel, Pantaleoni, Clark, Patten, McFarlane, Hobson, etc., to frame a more satisfactory theory of value by combining the analysis of cost and of marginal utility, are reviewed.

Mj. Autumn Quarter; 8: 30.
Assistant Professor Davenport.

  1. Labor and Capital. — Unsettled problems of distribution. The more abstruse questions of distribution will be considered. No student, therefore, can undertake the work of this course with profit who has not already become familiar with the fundamental principles. The course is open only to those who have passed satisfactorily Course 2, or who can clearly show that they have had an equivalent training. The subjects to be considered will be as follows: The wages-fund and other theories of wages, the interest problem, managers’ profits, and allied topics. The discussion will be based upon selected passages of important writers. The study of wages, for example, will include reading from Adam Smith, Ricardo, J. S. Mill, Longe, Thornton, Cairnes, F. A. Walker, Marshall, George, Böhm-Bawerk, Hobson, J. B. Clark, and others. Students will also be expected to discuss recent important contributions to these subjects in current books or journals.

Mj. Winter Quarter; 12:00.
Professor Laughlin.

  1. Economics of Workingmen. — Continuing the study of distribution (Course 41), examination is here undertaken of social movements for improving the condition of labor, to determine how far they are consistent with economic teaching, and likely in fact to facilitate or to retard economic betterment of workingmen. Efforts to increase earnings through modification of the wages system itself, resort to legislation, and the purposes and practices of labor organization are discussed, and the effect upon labor efficiency, earning capacity and steadiness of employment, of modern industrial systems; workingmen’s insurance; co-operation; profit-sharing; competition of women and children; industrial education; social-settlement work; consumers’ leagues. Interest centers about practical efforts for economic amelioration of employment conditions in “sweated” and in other industries. These studies are supplemented by statistical data on the condition of labor in different countries.

Mj. Winter Quarter; 9:30.
Assistant Professor Cummings.

Note. — Although open in certain cases to students of Sociology and others who have had the equivalent of the economic Courses 1 and 2, this course can be taken to best advantage by those only who have already had Course 41.

  1. Socialism — A history of the growth of socialistic sentiment and opinion as shown in the socialistic movements of the nineteenth century, and the position occupied by socialistic organizations of the present time. The course is in part historical and descriptive, in part theoretical and critical. The programmes and platforms of various socialistic organizations are examined and compared, and the theories of leading socialists are taken up in detail. Marx is given the chief share of attention, but other theoretical writers, such as Rodbertus, Kautsky, Bernstein, are also reviewed. The factors which at the present time further or hinder the spread of socialism, and what are its chances of being carried through or of producing a serious effect upon the institutions of modern countries, are considered.

Mj. Spring Quarter; 9:30.
Assistant Professor Veblen.

  1. Organization of Business Enterprise—Trusts. — A discussion of the growth of the conditions which have made large business coalitions possible, the motives which have led to their formation, the conditions requisite to their successful operation, the character and extent of the advantages to be derived from them, the drawbacks and dangers which may be involved in their further growth, the chances of governmental guidance or limitation of their formation and of the exercise of their power, the feasible policy and methods that may be pursued in dealing with the trusts. The work of the course is in large part investigation of special subjects, with lectures and assigned reading.

Mj. Autumn Quarter; 9:30.
Assistant Professor Veblen.

  1. Trades Unions and the Labor Movement — An historical and comparative study of the trades union movement in the United States and in foreign countries. Negotiation and maintenance of wage-compacts; methods of arbitration, conciliation and adjustment; trades union insurance and provision for the unemployed; incorporation and employés’ liability; the precipitation and conduct of strikes; and in general all concrete issues involved in the organization of labor for collective bargaining with employers, with especial reference to the working programs of the more important trades unions at the present time.

Summer Quarter; 9:00.
Mj. Spring Quarter; 12:00.
Assistant Professor Cummings.

  1. The Industrial Revolution and Labor Legislation. — The social consequences to the wage-earner of the development of the factory system of industry and of industrial development, more particularly during the last half of the 19th century, are taken up historically and descriptively. The social status of the modern wage-earner is contrasted with that of the handicraftsman working under more primitive conditions, and especial attention is given to the development of the modern wages system of remuneration, the historical modification of the labor contract in its legal aspects, and, finally, to the course of labor legislation which has in different countries accompanied industrial reorganization and development.

Mj.
Assistant Professor Cummings.

[Not to be given in 1904-5.]

Group V — Money and Banking.
  1. Money and Practical Economics.— An examination is first made of the principles of money, whether metallic or paper; then either the subject of metallic or paper money is taken up and studied historically, chiefly in connection with the experience of the United States, as a means of putting the principles into practice. Preliminary training for investigation is combined in this course, with the acquisition of desirable statistical information on practical questions of the day. The student is instructed in the bibliography of the subject, taught how to collect his data, and expected to weigh carefully the evidence on both sides of a mooted question. The work of writing theses is so adjusted that it corresponds to the work of other courses counting for the same number of hours.

Mj. Autumn Quarter; 12:00.
Professor Laughlin.

  1. The Theory and History of Banking. — A study is made of the banking systems of leading nations; the relations of the banks to the public; their influence on speculation; and the relative advantages of national banks, state banks, trust companies, and savings banks.

Mj. Winter Quarter; 8:30.
Mr. ———.

  1. Advanced Course in Money. — After having been drilled in the general principles of money (Course 50) the student is given an opportunity to examine the more difficult problems of money and credit.

Mj. Spring Quarter.
Professor Laughlin.

[Not given in 1904-5.1

  1. Practical Banking. — The internal organization and administration of a bank; the granting of loans; the valuation of an account; bank records; arithmetic of bank operations; mechanical and other time-saving devices.

Mj. Spring Quarter; 8:30.
Mr. ———.

  1. Commercial Crises. — A practical study of the operations of credit in the experiences of this and other countries during the periods of crises.

Mj. Spring Quarter.
Mr. ———.

[Not given in 1901-5.]

Group VI — Railways.
  1. Railway Transportation. — The economic, financial, and social influences arising from the growth of modern railway transportation, especially as concerns the United States, will be discussed. An account of the means of transportation developed in Europe and America during the early part of this century; the experiments of the states in constructing and operating canals and railways; national, state, and municipal aid to private companies; the rapid and irregular extension of the United States railway system; the failures of 1893; the reorganizations and consolidations since that time, with some attention to railway building in other countries, will form the historical part of the work. A discussion of competition, combination, discrimination, investments, speculation, abuse of fiduciary powers; state legislation and commissions, and the Inter-State Commerce Act, with decisions under it; and the various relations of the state, the public, the investors, the managers and the employés, will form the most important part of the work. This course gives a general view of the subject. Students who wish to continue the work by investigating special problems will have an opportunity to do so under Courses 61 and 62.

Mj. Autumn Quarter; 2:00
Assistant Professor Hill.

  1. The Regulation of Railway Rates. — The efforts of the railways of the United States to regulate railway rates through pools, will be compared with the efforts of the several states, and of the federal government, to regulate rates through legislation and through commissions. Typical decisions of pools, of state commissions, and of the Interstate Commerce Commission, will be studied for the purpose of ascertaining: (a) whether the decisions of the commissions are founded on a body of principles that may be said to have the character of a science, or, whether they express merely the judgment of administrative officers on questions of fact to which no body of scientific principles can be made to apply; (b) whether the past experience warrants the faith that the public regulation of railway rates will leave the railways sufficiently unhampered to develop trade and industry; (c) whether regulation by public authority promises to achieve more substantial justice than regulation by pools. The experience of Germany, France, Austria-Hungary, and Russia with the public regulation of railway rates — exercised either by legislation or by public ownership — will be studied with reference to the effect of such regulation upon the elasticity of railway rates, and upon the ability of the railways to develop trade and industry. In this connection will be studied the part played respectively by the railways and by the waterways in the development of Germany, France, Austria-Hungary, and Russia. The study will show why the countries in question are obliged to have recourse to the waterways for services that, in the United States, are rendered by the railways.

Mj. Winter Quarter; 2:00.
Assistant Professor Meyer.

  1. Industrial Activities of the State in Europe. — This course reviews the efforts made in Great Britain to secure to the public a share in the profits to be made in those so-called public service industries that use the streets: water, gas, electric light, street-railways, and hydraulic power, or compressed air, power transmission. These efforts consist of the imposition of severe restrictions upon franchises, with the alternative of municipal ownership. The experience of Great Britain will be compared with that of the United States, under: (a) the practice of practically no restrictions upon the industries in question; (b) the Massachusetts practice of regulation by legislation which is enforced and supplemented by state commissions. As for Continental Europe, the course will cover the experience of Prussia, France, and Russia, in attempting to make the railway and public works budgets fit into the state budget. In this connection the inelasticity of state activity in Europe will be compared with the elasticity of private activity in the United States.

Mj. Winter Quarter; 3:00.
Assistant Professor Meyer.

  1. The Industrial Activities of the State in Australasia. — This course will cover the Australasian experience of the last forty years under a wide extension of the functions of the state. Although Australasia is a comparatively small country, the experience in question is more significant than might appear at first sight, for it is the experience of a homogeneous, English-speaking people. The course will cover the management of the state-railways; the administration of the public finances; the civil service; and the legislative regulation of the conditions of labor, such as the fixing of minimum wages, and the establishment of compulsory arbitration. Incidentally comparisons will be made with certain conditions and practices in Great Britain and France, for the purpose of showing how the extension of the functions of the state has made the politics of Australasia resemble, in many vital respects, the politics of France, rather than those of Great Britain.

Mj. Spring Quarter; 2:00.
Assistant Professor Meyer.

  1. American Competition in Europe since 1873. — This course is a study in economics and politics; it purposes to put before the student information equipping him for the critical consideration of the merits of the question: Laissez faire vs. state intervention. To that end it institutes a series of comparisons between the United States and Europe, especially in the fields of agricultural practice and railway transportation. The course begins with the consideration of the nature of the competition to which the opening of new sources of supply of food products exposed Western Europe, the nature of the adjustments demanded by the situation, and the adjustments actually achieved, under free-trade in Great Britain, and under protection on the Continent. The course then proceeds to contrast the comparative failure to develop the agricultural resources of Eastern Europe (the Danubian Provinces and Russia) and Siberia with the rapid development of the agricultural resources of the interior regions of the United States. In this connection will be studied the comparative efficiency of the railway systems of Europe and the United States, with especial reference to the effect of the public regulation of railway rates, either through state-ownership, or through legislative and administrative intervention. Incidental to the main investigation an array of facts will be presented bearing upon questions of economic theory: the growth of population and the raising of the standard of living; some of the principal factors that have determined the present scale of real wages in the several European countries; some instances of the working of natural selection; and the relative merits of large farms and small farms, or of extensive cultivation and intensive cultivation.

Mj. Spring Quarter; 3:00.
Assistant Professor Meyer.

Group VII — Statistics.
  1. Training Course in Statistics. — The object of this course is to train students in the practical use of statistical methods of investigation. Stress is laid upon work done by students themselves in collecting, tabulating, interpreting, and presenting statistics of different orders. Members of the class are also required to make close critical examinations of various publications of statistical nature with a view to determining the accuracy of data and the legitimacy of inferences drawn. Students engaged in any special work of investigation are encouraged to deal mainly with data relevant to their subjects. To others special topics are assigned. It is hoped that the course may prove useful to all students whose work, in whatever department it may lie, whether in history, sociology, or in other fields of study, is susceptible of statistical treatment.
    Courses 70 and 72 will be given in alternate years.

Mj. Autumn Quarter; 8:30.
Assistant Professor Cummings.

  1. Statistics of Wages in the Nineteenth Century. — In this course effort is made to determine what has been the actual movement of wages during the nineteenth century. An examination is undertaken of the more important statistical investigations of wage movements which have been made from time to time by economists, government bureaus, or other agencies, in specific industries; the object being to determine the extent to which the wage-earner has in general participated in the benefits of industrial progress and of the increased economic efficiency of labor and capital. The course is intended to be informational and descriptive in character, as well as to give training in the collection and tabulation of statistical data.

Mj. Winter Quarter; 12:00.
Assistant Professor Cummings.

  1. Demography. — Statistical methods are illustrated by studies in population data, comprising the construction of actuarial tables; determination of the economic value of populations; economic aspects of the data of criminality and pauperism; growth and migration of population in the United States as “labor force,” including statistics of the negro race. The development of official statistics of population, and the demographic work of government bureaus is taken up historically and critically. The object of the course is to give students training in handling population data as a basis of sociological and economic speculation, and to point out the bearing of such data and their importance in the historical development of economic theories.

Assistant Professor Cummings.

[Not to be given in 1904-5.]

Group VIII — The Seminars.

80, 81. Economic Seminar.

2Mj. Autumn and Winter Quarters.
Professor Laughlin.

Source: University of Chicago. Programme of the Departments of Political Economy, Political Science, History, Sociology and Anthropology, 1904-1905. Chicago: The University of Chicago Press, 1904. Transcription from a copy found in the Harvard University Archives, Division of History, Government, and Economics. Ph.D. exams and records of candidates, study plans, lists, etc. pre-1911-1942. Box 2, Unlabeled Folder.

Image Source: Technology Reading Room 2, Crerar Library (Marshall Field Annex). From the University of Chicago Photographic Archive, apf2-01949, Hanna Holborn Gray Special Collections Research Center, University of Chicago Library.

Categories
Columbia Economists Methodology New School

Columbia. Wesley Clair Mitchell’s remarks at Conference on Methods in Philosophy and the Sciences, 1937

 

In brief remarks intended to give non-economists a sense of the major methodological schools of economics at a 1937 conference at the New School for Social Research, Columbia professor Wesley Clair Mitchell distinguishes (i) orthodox economics dedicated to the understanding of the “pecuniary logic” of an agent within a capitalist market environment, (ii) institutional economics dedicated to the understanding of the evolution of economic organization, and (iii) a new, yet unnamed, type of economic theory that is clearly recognizable as being “behavioral economics”.

____________________

Conference Program

CONFERENCE ON METHODS IN PHILOSOPHY AND THE SCIENCES

New School for Social Research
66 West 12th Street
New York City, N.Y.

Saturday, May 22 and Sunday, May 23, 1937

PROGRAM

Saturday, May 22

* * * * * * * * * * * * * * * *

9:30 A.M. – 11:00 A.M – Registration, Room 24, Fee – $1.00

11:00 A.M. – 1:00 P.M. – First Session – Room 25

Chairman:  H. M. Kallen
Sidney Hook: The Current Philosophical Scene
John Dewey: A Possible Program for Libertarians and Experimentalists

* * * * * * * * * * * * * * * *

2:30 – 4:00 P:M: – Second Session – Room 25

Brief statements on various departments of philosophy and the sciences: Their assumptions, methods, histories of the different schools, etc.

Ernest Nagel: The Position in LOGIC and METHODOLOGY
W.M. Malisoff: The Position in the PHYSICAL SCIENCES

DISCUSSION

4:00 – 5:30 P.M. – Second Session Continued – Room 25

S. E. Asch: The Position in PSYCHOLOGY
Wesley C. Mitchell: The Position in ECONOMICS

DISCUSSION

* * * * * * * * * * * * * * * *

7:00 P.M. – DINNER, Gene’s 71 West 11th Street

Speakers: Bacchus, Dionysus, the Holy and other Spirits.
Appointment of Committees

* * * * * * * * * * * * * * * *

Sunday, May 23

10:00 A.M. – 12 M. – Third Session, Room 25

Julius Lips: The Position in ANTHROPOLOGY
Meyer Schapiro: The Position in AESTHETICS
R. M. MacIver: The Position in SOCIOLOGY

DISCUSSION

12M. – 1:00 P.M. – Business Meeting

Election of Officers
Appointment of Permanent Committees
Unfinished Business
Adjournment

 

____________________

Handwritten Remark by Wesley Clair Mitchell
Economics

 

Conference on Methods in Philosophy and the Social Sciences

New School
May 22, 1937

Economics like Philosophy and the other Social Sciences is still in the stage of development marked by the existence of fairly distinct schools of thought, or as I like better to say Types of Theory.

These schools differ in method. But these differences in method arise from differences in the problems which are taken as the central concern of economics.

 

Orthodox economics concerns itself primarily with what I like to call pecuniary logic — what it is to the economic advantage of men to do under a capitalistic organization — and the ‘purer’ this theory becomes the more exclusive concentration on that problem becomes.

In dealing with pecuniary logic, the investigator employs the method of imaginary experimentation. That is, he sets up certain assumptions and seeks to think out what it is to the interest of men to do under the conditions supposed.

The theory is developed by varying these assumptions with reference to such matters as the factors in theory set which are allowed to change the length of the period considered in the problem, the degree of competition supposed, elasticities of demand, relations between unit costs and volume of output.

How far the conclusions apply to the actual world depends upon the character of the assumptions made. The correspondence between these assumptions and actual conditions is seldom investigated.

Hence the doubts about this type of theory are usually doubts, not about the correctness of the reasoning, but about how far they apply to the facts we wish to understand. May have uncertain ‘operational significance’.  Defence.—tool makers. Question about applicability not relevant.

This description applies less strictly to Marshall than to many of his pupils, to the later Austrians, and to mathematical economists.

 

Institutional economics concerns itself primarily with the evolution of economic organization.

To Veblen this meant study of the widely prevalent habits of thought.

To Commons it means study of social controls over induced action—primarily through the courts.

Methods employed combine ethnology or historical research with reasoning about how men with a certain set of habits ingrained in them by the social environment in which they have grown up and by the work they do will behave or how the social controls over induced behavior may be expected to work out.

Again there may be doubts about how far the reasoning concerning economic behavior applies to actual conditions.

 

A third type of economics seems to be developing though not represented as yet by systematic theoretical treatises.

It endeavors to learn by analytic studies of actual behavior how men conduct themselves. Its methods are closer kin to those of animal psychologists than to those of introspective psychologists.

Though these men show no reluctance to account for their observations by supposing that their subjects know the rules of the money-making and money-spending games. Here they go beyond outlook[?] of physical science— Supposes men have purposes: that they plan for future .

Large use of the mass observations afforded by statistics

Considerable emphasis upon method[?] analysis of these records.

Not confined to statistics.

Doubts here concern representative value [or volume?] of the data

Trustworthiness of the mathematical analysis.

Extent to which factors that are not recorded statistically may modify conclusions drawn. Work of this sort is primarily monographic. Since social phenomena are interdependent, the question concerning what is left out is highly important

Can’t be applied well except when mass observations are available.

Promises to develop in future because statistical observation is covering a wider range.

Danger of ‘mere fact finding’ Dewey. Yes, but the facts may have deep ‘operational’ significance. Relation to questions of policy.

 

Source: Columbia University Archives. W. C. Mitchell Collection. Box 3, Folder “5/22/37 A”.

Image Source: Wesley Clair Mitchell from Albert Arnold Sprague’s and Claudia C. Milstead’s Genealogical Website.

 

Categories
Exam Questions Swarthmore Undergraduate

Swarthmore. Senior comprehensive economics exam, 1931

 

The two previous posts provided undergraduate comprehensive examinations for Harvard and Princeton from the early 1930s that were published in the Bulletin of the Association of American Colleges (December, 1933). The cross-section of comprehensive economics exams is now expanded with this post to include Swarthmore College’s economics department.

A decade later Swarthmore College brought in external examiners (many of whom recruited from Wolfgang Stolper’s network of Harvard graduate buddies), e.g.

_________________

Senior Comprehensive Examination in Economics

Swarthmore College, 1931

  1. a. Why have railroads been subject to an unusual amount of regulation?
    b. Appraise reproduction cost as a basis for the valuation of public utilities.
    c. Explain the operation of the principle of joint costs in the determination of rates for specific services.
  2. Discuss:
    a. “One of the unions’ chief errors is restriction of output, which is always against the social interest and even fruitless for the workers.”
    b. “To the extent that employee representation seems to the worker to be just an employer’s weapon against trade unionism, it will be still less popular in the future than today and even its good points will be ignored.”
  3. Is it necessary to make goods in order to make money? Give the answers of T. N. Carver, S. & B. Webb, F. M. Taylor, T. Veblen, Adam Smith, R. H. Tawney, and Alfred Marshall. Why have these scholars come to such contrary conclusions after examining the facts? Is it possible that both groups are right; that neither one is right? How so? If not, which group is right and why?
  4. a. Since it is understood that all kinds of money in this country are to be maintained at a parity of value with standard money, would not inconvertible paper money issued by our government be quite as acceptable and useful as any kind? Explain.
    b. In the United States there are many kinds of money. What are they, and what is the security behind each one? Does Gresham’s Law operate? Why, or why not.
  5. a. It is said that the United States is evolving into a commission form of government; that the government set up by the Constitution is gradually delegating its duties to “expert commissions.” Bearing in mind the frailty of commissioners and their staffs, do you believe this is a wise movement? Why? Be specific.
    b. Giving generous reference to the history of governmental regulation in the United States, what do you believe will be the position of the government as a regulator of business twenty-five years hence?
  6. According to present estimates, the federal government will complete this fiscal year, June 30, 1931, with a deficit of nearly one billion dollars. Outline, in detail, the causes of this deficit. Suggest, with reasons, the fiscal program which the government should adopt, for the coming year, in view of this deficit.
  7. Philip Snowden, British Chancellor of the Exchequer, has proposed the imposition of a tax on the site value of land as a means of balancing the British budget. It is argued that such a tax would have less of a repressive effect upon industry than any other type of tax which might be imposed. Give in detail the reasoning which supports this position.
  8. Give an historical account of the currency agitation and legislation from the end of the Civil War to the end of the last century. What issues were involved and how did they arise? On the whole, do you think that our currency history of this period refutes or verifies the quantity theory of money?
  9. a. Imagine yourself a Congressman in the year of 1828 and make a brief argument for the high tariff policy adopted in that year. Would you argue in the same way today? If not, why not? Is your supposed speech that of a representative from South Carolina or Pennsylvania? Give reasons.
    b. Briefly comment upon what you regard as three important causes or factors in the present industrial depression.
  10. a. How important for price theory and for practical life are differences in the elasticity of demand for commodities? Illustrate, using diagrams.
    b. Translate, and if necessary, correct the following popular statements into the more exact language of economic theory:

(1) “We produce too much coal and people freeze to death; we raise too much cotton and people go naked.”
(2) “Great Britain’s foreign trade is in a bad way; she has an extremely unfavorable balance.”
(3) “The price of corn is low because you can buy good corn land so cheaply.”
(4) “Depressions are due to over-production, and by this I mean that more goods are produced than can be sold, for two reasons; rich people save too much, and the workers do not get high enough wages.”

(Answer five questions. Use the first half-hour to study and select your questions. Then devote about thirty minutes to answering each question.)

 

Source:  Edward S. Jones. Comprehensive Examinations in the Social Sciences, Supplement to the December, 1933 Bulletin of the Association of American Colleges, pp. 41-43.

Image Source: Parrish Hall, Swarthmore College  .

 

Categories
Chicago Curriculum Economics Programs

Chicago. First detailed announcement of Political Economy program, 1892

 

The founding Head-Professor of Political Economy, J. Laurence Laughlin, arrived at the University of Chicago in June 1892. The following printed announcement of the programme of courses in political economy ends with a call for fellowship applications with a last-submission date of June 1, 1892. Thus we can presume that Laughlin had organized his department’s course offerings and staffing before he physically reported for duty,  and I would guess this announcement was published in the late winter/early spring of 1892. Besides being an artifact we can date to the Big Bang moment of creation of the University of Chicago’s department of political economy, the announcement provides us relatively thick descriptions of the respective courses in the political economy programme at that early date.

 

_______________

THE UNIVERSITY OF CHICAGO
1892-3

PROGRAMME OF COURSES IN POLITICAL ECONOMY

CHICAGO
The University Press of Chicago
1892

*  *  *  *  *  *  *

DEPARTMENT OF POLITICAL ECONOMY.
1892-3.

OFFICERS OF INSTRUCTION:

J. LAURENCE LAUGHLIN, Ph. D., Head-Professor of Political Economy.

ADOLPH C. MILLER, A. M., Associate-Professor of Political Economy.

WILLIAM CALDWELL, A. M., Tutor in Political Economy.

*  *  *  *  *  *  *

INTRODUCTORY.

The work of the department is intended to provide, by symmetrically arranged courses of instruction, a complete training in the various branches of economics, beginning with elementary work and passing by degrees to the higher work of investigation. A chief aim of the instruction will be to teach methods of work, to foster a judicial spirit, and to cultivate an attitude of scholarly independence, (1) The student may pass, in the various courses of instruction, over the whole field of economics; (2) when fitted, he will be urged to pursue some special investigation. (3) For the encouragement of research and the training of properly qualified teachers of economics, Fellowships in Political Economy have been founded. (4) To provide a means of communication between investigation and the public, a review, entitled The Journal of Political Economy, has been established, to be edited by the officers of instruction in the department; while (5) larger single productions will appear in a series of bound volumes to be known as Economic Studies of the University of Chicago.

REMARK: In the following list the term Minor, is applied to a course which calls for four or five hours of class-room work per week for a period of six weeks. A Double Minor is a Minor running through two periods of six weeks.

 

LIST OF COURSES OF INSTRUCTION.

STARRED * COURSES ARE NOT GIVEN IN 1892-3.

  1. First Quarter: Principles of Political Economy. — Exposition of the Laws of Political Economy in its present state. — Mill’s Principles of Political Economy (Laughlin’s edition).

5 hrs. a week, Double Minor.
Associate Professor A. C. Miller.

Second Quarter:

Either, 1A. Advanced Political Economy. — Cairnes’s Leading Principles of Political Economy. — Marshall’s Principles of Economics (vol. I.).

5 hrs. a week, Double Minor.
Mr. Caldwell.

Or, 1B. Descriptive Political Economy. — Lectures and Reading on Money, Banking, Cooperation, Socialism, Taxation, and Finance. — Hadley’s Railroad Transportation. — Laughlin’s Bimetallism.

4 hrs a week, Double Minor.
Associate Professor A. C. Miller.

  1. Industrial and Economic History. — Leading Events in the Economic History of Europe and America since the middle of the Eighteenth Century. — Lectures and Reading.

4 hrs. a week, 2 Double Minors.
Mr. Caldwell.

  1. Scope and Method of Political Economy. — Origin and Development of the Historical School. — History of Political Economy in Germany. — Lectures and Reports.

4 hrs. a week, Double Minor.
Mr. Caldwell.

  1. Unsettled Problems of Economic Theory. — Questions of Exchange and Distribution.— Critical Examination of selections from leading writers.

4 hrs. a week, Double Minor.
Professor Laughlin.

  1. History of Political Economy. — History of the Development of Economic Thought, embracing the Mercantilists and the Physiocrats, followed by a critical study of Adam Smith and his English and Continental Successors. — Lectures and Reading. — Reports.

5 hrs. a week, Double Minor.
Mr. Caldwell.

  1. Recent German Systematic Writers*. — Wagner, Cohn, Schmoller, Schäffle, and Menger. — Exposition, critical comments, and reading of authors. — Reports.

4 hrs. a week, Double Minor.
Mr. Caldwell.

  1. Socialism. — History of Socialistic Theories. — Recent Socialistic Developments. — Lectures and Reports.

4 hrs. a week, Double Minor.
Dr. Veblen.

  1. Social Economics. — Social questions examined from the economic standpoint.

A*. Social Reforms. — Future of the Working-classes. — Immigration. — State Interference. — Insurance Legislation. — Arbeitscolonien.

4 hrs. a week, Double Minor.
Mr. Caldwell.

B. Coöperation. — Profit-Sharing. — Building Associations. — Postal Savings. — Trade Unions. — Factory Legislation. — Public Charities.

4 hrs. a week, Double Minor.
Associate Professor Bemis.

  1. Practical Economics. — Training in the Theoretical and Historical Investigation of Important Questions of the Day. — Lectures and Theses.

4 hrs. a week, Double Minor.
Associate Professor A. C. Miller.

  1. Statistics. — Methods and practical training. — Organization of Bureaus. — Tabulation and Presentation of Results.

4 hrs. a week, Double Minor.
Mr. Fisher.

  1. Railway Transportation. — History and Development of Railways. — Theories of Rates. — State Ownership.

4 hrs. a week, Double Minor.
Professor Laughlin.

  1. Tariff History of the United States. — Legislation since 1789. — Economic Effects. — Reading.

4 hrs. a week, Double Minor.
Professor Laughlin.

  1. Financial History of the United States.— Rapid Survey of the Financial Experiences of the Colonies and the Confederation. — Detailed Study of the Course of American Legislation on Currency, Debts, and Banking since 1789. — Lectures and Reports.

4 hrs. a week, Double Minor.
Associate Professor A. C. Miller.

  1. Taxation. — Theories and Methods of Taxation. — Comparative Study of the Revenue Systems of the Principal Modern States. — Problems of State and Local Taxation in America. — Lectures and Reports.

4 hrs. a week, Double Minor.
Associate Professor A. C. Miller.

  1. Public Debts and Banking. — Comparative Study of European and American Methods of Financial Administration. — The Negotiation, Management, and Effects of Public Debts. — Examination of Banking Problems and Banking Systems. — Lectures and Reports.

4 hrs. a week, Double Minor.
Associate Professor A. C. Miller.

  1. Problems of American Agriculture*. — Comparison with European Systems of Culture. — Land Tenures. — Lectures, Reading, Reports.

4 hrs. a week, Double Minor.
Professor Laughlin.

  1. Seminary. — Intended only for mature students capable of carrying on independent researches.

4 hrs. a week, 3 Double Minors

 

 

DESCRIPTION OF COURSES
GENERAL.

The courses may be roughly classified into —

Group I., Elementary. — Courses 1, 1A, 1B, and 2;
Group II., Theoretical. — Courses 3, 4, 5, 6, and 7;
Group III., Practical. — Courses 8, 9, 10, 11, 12, 13, 14, 15, and 16.

Students are advised to begin the study of economics not later than the first year of their entrance into the University College; and students of high standing, showing special aptitude for economic study, may properly take Course 1 in the last year of the Academic College.

For admission into the courses of Groups II. and III., a prerequisite is the satisfactory completion of Course 1 (with either 1A or 1B), or its equivalent. Those desiring only a general acquaintance with the subject are expected to take Course 1B during the second quarter; but those who intend to make a serious study of economics are advised to take 1A during the second quarter.

After passing satisfactorily in Course 1 (with either 1A or 1B), the student will find a division of the courses into two general groups: Group II. will be concerned chiefly with a study of economic principles, their historical development, and the various systems of economic thought; Group III., while making use of principles and economic reasoning, will be devoted mainly to the collection of facts, the weighing of evidence, and an examination of questions bearing on the immediate welfare of our people. For a proper grasp of the subject, Courses 3, 4, and 5 are indispensable; and in the second year of his study of economics the student should supplement a course in Group I. by a course in Group II.

Ability to treat economic questions properly can be acquired only if the student, being possessed of some natural aptitude for the study, devotes sufficient time to it to enable him to assimilate the principles into his thinking, and to obtain certain habits of mind, which are demanded for proficiency in this, as in any other important branch of study. Tests of proficiency will be exacted at the end of each period, six weeks.

 

 

SPECIAL.

COURSE 1.

First Quarter: Principles of Political Economy. —Exposition of the Laws of Political Economy in its Present State. — Mill’s Principles of Political Economy (Laughlin’s edition).

5 hrs. a week, Double Minor.

Second Quarter:

Either, 1 A. Advanced Political Economy.— Cairnes’s Leading Principles of Political Economy. — Marshall’s Principles of Economics (vol. I).

5 hrs. a week, Double Minor.

Or, 1B. Descriptive Political Economy. — Lectures and Reading on Money, Banking, Cooperation, Socialism, Taxation, and Finance. — Hadley’s Railroad Transportation. — Laughlin’s Bimetallism.

4 hrs. a week, Double Minor

All students beginning the study of Political Economy will take Course 1. At the second quarter the class will divide. Those desirous of laying the foundation for work in the advanced courses will take 1A; those who, while giving their attention mainly to other departments, seek simply that general knowledge of economics demanded by a liberal education, and cannot devote more time to the study, will take Course 1B. Course 1 is designed to give the student an acquaintance with the working principles of Political Economy.

Course 1A will continue the theoretical training in the principles of Political Economy. The discussions will be based on Cairnes’s Leading Principles of Political Economy, and Marshall’s Principles of Economics (vol. I). Only those students who have passed satisfactorily in Course 1A, will be admitted to Courses 3, 4, 5, 6, 7 and 9.

Course 1B is mainly descriptive and practical; in it will be considered the various practical questions illustrating the application of economic principles, the lectures and reading supplying the student with the knowledge necessary for the more intelligent discharge of the duties of citizenship. The subjects discussed will be: Money, banking, coöperation, socialism, taxation, finance, and railway transportation. Students will be expected to read Hadley’s Railroad Transportation, and Laughlin’s History of Bimetallism in the United States.

COURSE 2.

Industrial and Economic History. — Leading Events in the Economic History of Europe and America since the middle of the Eighteenth Century. Lectures and Reading.

4 hrs. a week, 2 Double Minors.

This course endeavors to present a comprehensive survey of the industrial, commercial, and economic development of the western world since the middle of the last century. After a preliminary study of the industrial revolution and the rise of the factory system, attention will be called to the economic and social effects of the American and French revolutions; to the development of American commerce; to the introduction of steam transportation; to the adoption of free trade by England; to the new gold discoveries and their wide-spread effects; to the civil war in the United States; to the French indemnity; to the crisis of 1873; and to the economic disturbances of the past twenty years. The course is conducted mainly by lectures, but a course of collateral reading will be prescribed upon which students will be expected to report from time to time.

No previous economic study is required of students entering this course, but it will be taken to best advantage by those who already have some knowledge of economic principles, or who are taking this course in connection with Course 1.

COURSE 3.

Scope and Method of Political Economy. — Origin and Development of the Historical School. — History of Political Economy in Germany. Lectures and Reports.

4 hrs. a week, Double Minor.

This course attempts to define the province, postulates and character of Political Economy; to determine its method, and to examine the nature of economic truth. The methods of proof and the processes of reasoning involved in the analysis of economic phenomena and the investigation of economic problems, and the position of Political Economy in the circle of the Moral Sciences — its relation to Ethics, Political Science, and Sociology — will be studied. In view of the controversies which have arisen on these fundamental topics, a critical estimate will be made of the views of leading writers on Methodology, such as Mill, Cairnes, Menger, Wagner and Schmoller.

The origin and development of the modern historical school will be described, special attention being devoted to Knies, Die Politische Oekonomie vom Geschichtlichen Standpunkte.

In connection with this work, the course of German economic thinking will be traced from the earlier writers, Rau, von Thünen, and Hermann; after which the influence of the English writers, the later formation of various groups, with their distinguishing tenets, and the German point of view, will be presented. The statements of the writers themselves, rather than opinions about them, will be studied.

Students will be required to prepare critical studies on books, or subjects, selected by the instructor. Course 3 is preliminary to Course 6.

COURSE 4 .

Unsettled Problems of Economic Theory. — Questions of Exchange and Distribution. Critical examination of selections from leading writers.

4 hrs. a week, Double Minor.

Little use will be made of text-books, or lectures, in this course, it being intended to take up certain topics in economic theory and to follow out their treatment by various writers. The more abstruse questions of exchange and distribution will be considered. No student, therefore, can undertake the work of this course with profit who has not already become familiar with the fundamental principles. The course is open only to those who have passed satisfactorily in Course 1A., or who can clearly show that they have had an equivalent training.

The subjects to be considered in 1892-3 will be as follows: The theories of final utility and cost of production as regulators of value, the wages-fund and other theories of wages, the interest problem, manager’s profits, and allied topics. The discussion will be based upon selected passages of important writers. The study of wages, for example, will include reading from Adam Smith’s Wealth of Nations, Ricardo’s Works, and the writings of J. S. Mill, Longe, Thornton, Cairnes, F. A. Walker, Marshall, George and Böhm-Bawerk. Students will also be expected to discuss recent important contributions to these subjects in current books or journals; and they will be practised in the exposition of special points before their fellow students.

COURSE 5 .

History of Political Economy. History of the Development of Economic Thought, embracing the Mercantilists and the Physiocrats, followed by a critical study of Adam Smith and his English and Continental Successors. Lectures, Reading, and Reports.

5 hrs. a week, Double Minor.

This course treats of the history of economic theory, not of the history of economic institutions; of the origin and development of our existing knowledge of economic principles, not of the phenomena of wealth with which these economic principles are concerned. Since it investigates the evolution of economic thinking as expressed in a growing collection of principles, the student will have little occasion to study writers previous to the XVI century. The time will be given to the economic theories and commercial policy of the Mercantile system; to the Physiocratic school; to Adam Smith and his immediate precursors; to the English writers from Adam Smith to the present day; and to a brief review of French, Italian, and American writers. From the multiplicity of writers, selections will be made of those who have had great influence, or who have made marked contributions to political economy. The whole study will aim to present the continuity of development of economic doctrine from its origin to the present time.

The work, however, is not intended merely as a means of information. It is expected that the student himself should in every case read portions of the great authors bearing on cardinal principles, and, by critical comment and comparison, it is hoped he may gain much in discipline and in judicial insight. It is believed that a more fresh, original, and just understanding of the history of political economy can be obtained by this mode of treatment than by taking a knowledge of the authors at second-hand. The work of this course, therefore, must largely be carried on in the Economic Library. In this, as in other courses, the instructor will pay early attention to bibliography and to the best methods of using books.

COURSE 6.

Recent German Systematic Writers. Wagner, Cohn, Schmoller, Schäffle, and Menger. Exposition, critical comments, and reading of authors. Reports.

4 hrs. a week, Double Minor.

It is the object of this course to present the point of view of the leading recent German writers in Economics, through a study of the character and contents of their systematic treatises. In this way it is hoped that the desire for a direct acquaintance with the particular economic doctrines which are actually taught in Germany at the present day can be adequately met. The student will be helped to appreciate the spirit, quality, and tendency of German economic thinking, and thus be enabled to broaden his view of fundamental economic ideas.

The instructor will outline the system of each writer, give the substance of less important portions, and, with comments, translate in the class-room considerable selections. The student will be expected to have a working knowledge of German, and will be required to read parts of the authors not read in the class-room, upon which reports and critical studies in writing must be made. An incidental aim of this course will be to assist the student in acquiring a rapid reading knowledge of economic German.

The authors to be used are as follows:

Wagner, Volkswirthschaftslehre. Grundlegung.
Schmoller, Ueber einige Grundfragen des Rechts und der Volkswirthschaft.
Schäffle, Bau und Leben des socialen Körpers (ed. 1881).
Cohn, System der Nationaloekonomie. Grundlegung.
Menger, Grundsätze der Volkswirthschaftslehre,

COURSE 7.

Socialism. History of Socialistic Theories. Recent Socialistic Developments. Lectures and Reports.

4 hrs. a week, Double Minor.

The origin of the present socialistic movements, whether popular or scientific, will be traced to their beginnings previous to the middle of the present century; the events ending in 1848 will be described; and an examination will be made of the writings of Rodbertus, Marx, Lassalle, Karl Marlo, and William Thompson, from the economic standpoint. The criticisms offered, among other writers, by Leroy-Beaulieu, Rae, H. Spencer, and Schäffle, will be brought under review. A study of the “International” will be followed by an account of the spread of Socialism to England and America. The position and tenets of the Fabian Society in England; the popular agitations of the present day in Europe and America; the socialistic tendencies imputed to George’s Progress and Poverty, Gronlund’s Coöperative Commomwealth, and Bellamy’s schemes for Nationalism, will be taken up. Practical work will be done with the programs and platforms of socialistic, labor, and trade organizations.

Attention will then be given to the alleged socialistic trend of development, to State Socialism, to the economic factors in operation, and to the ethical aspect of the economic questions involved.

Students will be expected to make written reports and critical studies from time to time, in addition to selected reading. Those who have not examined questions of value and distribution carefully will be at a disadvantage in this course.

COURSE 8.

Social Economics. Social questions examined from the economic standpoint.

Course 8 includes two separate courses, known as Course 8A, and Course 8B. Under these heads many subjects into whose treatment ethical and social considerations enter, but which have a distinct economic character, will be considered.

COURSE 8A. Economic Reforms — Future of the Working-classes. Immigration. — State Interference. — Insurance Legislation. — Arbeitscolonien.

4 hrs. a week, Double Minor.

Under this head the ethical, sociological and political principles underlying proposed practical reforms and methods of social improvement will be noticed and criticised, and their economic values and effects will be considered and estimated. It will be sought to determine, as far as possible, the teaching of history and experience on these matters, and also the conditions and range, the merits and defects of various experiments.

COURSE 8B. Coöperation. — Profit-Sharing. — Building Associations. — Postal Savings. — Trades Unions. — Factory Legislation. — Public Charities

4 hrs. a week, Double Minor.

In this course schemes of economic reform will be studied and presented with a view to inform the student how they may be carried out into actual practice. It is hoped that members of this course, under the guidance of the instructor, may be familiarized with the process of organizing desirable movements of a philanthropic character in various parts of the community.

Both of these courses may well be elected by candidates for the ministry, who have already passed in Course 1. Reading and reports will accompany the lectures.

COURSE 9.

Practical Economics.Training in the Theoretical and Historical Investigation of Important Questions of the Day. Lectures and Theses.

4 hrs. a week, Double Minor.

Preliminary training for investigation is combined in this course with the acquisition of desirable statistical information on practical questions of the day. The student is instructed in the bibliography of a subject, taught how to collect his data, and expected to weigh carefully the evidence on both sides of a mooted question. The short theses form a connected series, and give practice in written exposition as well as in the graphic representation of statistics. Mere compilation is objected to, and the student is urged to reach his conclusions independently and solely on the facts before him. Fresh and independent judgments are encouraged. The work of writing theses is so adjusted that it will correspond to the work of other courses counting for the same number of hours.

The instructor will criticise the theses before the class, and members of the class will be frequently called upon to lecture on the subjects of their theses and answer questions from their fellow-students.

The subjects taken up will be chosen from the following: Money, prices, bimetallism, note-issues, shipping, and commercial crises.

COURSE 10.

Statistics. Methods and practical training. Organization of Bureaus. Tabulation and Presentation of Results.

4 hrs. a week, Double Minor.

The purpose of this course is to train students in the theory and methods of statistics. Inasmuch as economic principles throw light upon the proper choice and comparison of statistical data, a knowledge of Course 1 is a prerequisite to entrance into this course. On the other hand, statistical methods are needed for the correction and furthering of our knowledge of economic principles.

Attention will be given to the vast statistical material at hand, and the student will have an introduction into the bibliography of the subject. The growth of the study; establishment of statistical offices and their organization; collection and elaboration of data; detection and elimination of errors; presentation of results in tabular form; training in graphic representation; — will form a part of the work.

Practical exercises will be required of each student in connection with the collection and presentation of statistics of mortality, insurance, production, population, wages, prices, trade, crime, etc. The great libraries of the City of Chicago will furnish exceptional advantages for this work.

COURSE 11.

Railway Transportation. History and Development of Railways. — Theories of Rates. — State Ownership.

4 hrs. a week, Double Minor.

The economic, financial, and social influences arising from the growth of modern railway transportation, especially as concerns the United States, will be discussed. The history of railway development in Europe and America; its social and economic influence; railway accounts; competition and combination; various theories of rates; railway legislation in the United States; state railway commissions; the Inter-State Commerce Act; government ownership; and a comparison with the railway systems of Great Britain, France, Germany, Austria, Italy, and Australia, — will form the essential work of the course.

Studies in writing will be exacted from each student. In addition to the lectures, the student is expected to read Hadley’s Railroad Transportation, and Acworth’s The Railways and the Traders.

COURSE 12.

Tariff History of the United States. Legislation since 1789. Economic Effects. Lectures and Reports. Reading.

4 hrs. a week, Double Minor.

Course 12 is fitly taken in connection with Course 13, which runs parallel with it. An historical study will be made of the legislation on the tariff in the United States from the beginning in 1789 to the present day. Study will be given to the provisions of each act, the causes of its passage and its economic effects. The growth of the principal industries of the country will be sketched in connection with the duties affecting them.

Students will be required to present studies on special topics connected with the course.

COURSE 13.

Financial History of the United States.— Rapid Survey of the Financial Experiences of the Colonies and the Confederation. — Detailed study of the course of American Legislation on Currency, Debts, and Banking since 1789. Lectures and Reports.

4 hrs. a week, Double Minor.

Without excluding the history of taxation, this course concerns itself chiefly with the history of our national legislation on currency, loans, and banking. The study will be based upon a careful examination at first-hand of the leading provisions of the Acts of Congress, and other materials important in our financial history. These will be reviewed from the political as well as from the financial standpoints, it being one of the objects of the course to develop the relation between finance and politics in our history. Special attention will be given to Hamilton’s system of finance and the changes introduced by Gallatin; to the financial policy of the War of 1812; to the establishment of the Second United States Bank and the struggles over its re-charter; to the crisis of 1837-9 and the establishment of the independent Treasury; to the financial problems and management of the Civil War; to the establishment of the national banking system; the refunding and reduction of the debt; and the resumption of specie payments.

COURSE 14.

Taxation. Theories and Methods of Taxation. — Comparative Study of the Revenue Systems of the principal modern States. — Problems of State and Local Taxation in America. Lectures and Reports.

4 hrs. a week, Double Minor.

This course is both theoretical and practical, and the method of presentation historical as well as systematic. A critical estimate of the theories of leading writers — such as Wagner, Cohn, Leroy-Beaulieu — will be made with a view of discovering a tenable basis of taxation. Principles are discussed; the various kinds of taxes are examined and their complementary functions in a system of taxes determined; the methods in vogue in different countries are described, special attention being given in this connection to the experiences of France. In their proper places the incidence of taxes, progressive taxation, the single tax, and the special problems of American taxation will be carefully considered. All questions will be discussed from the twofold standpoint of justice and expediency. A reading knowledge of either French or German will be expected of all students entering this course.

COURSE 15.

Public Debts and Banking. — Comparative Study of European and American Methods of Financial Administration. — The Negotiation, Management, and Effects of Public Debts. — Examination of Banking Problems and Banking Systems. Lectures and Reports.

4 hrs. a week, Double Minor.

This course treats of the organization and methods of financial administration; the formal control of public expenditures by means of the budget; the development of public debts and their economic and social effects. Consideration will be given to the various problems involved in the management of public debts, such as modes of issue, conversion, and reduction; and the methods practised in our own and other countries will be described. This course also treats of the development and history of banking; the leading systems are compared, and proposed changes in legislation examined. The relations of the banks to the public and their management in a time of crisis will receive special attention.

COURSE 16.

Problems of American Agriculture. Comparison with European systems of culture. Land Tenures. Lectures, Reading, and Reports.

4 hrs. a week, Double Minor.

Special study will be given to the extension and changes of the cultivated area in the United States; the methods of farming; the influence of railways and population, and of cheapened transportation; the fall in values of Eastern farm-lands; movements of prices of agricultural products; European markets; competition of other countries; intensive farming; diminishing returns; farm mortgages, and the comparison of American with European systems of culture. Systems of holdings in Great Britain, Belgium, France, and Germany will be touched upon, together with the discussion of forestry legislation.

Reports will be prepared by students on topics assigned.

COURSE 17.

Seminary. Intended only for mature students capable of carrying on independent researches.

4 hrs. a week, 3 Double Minors.

Under this head are placed the arrangements for Fellows, graduates, and suitably prepared persons, who wish to carry on special researches under the guidance of the instructors. Candidates for the higher degrees will find in the seminary a means of regularly obtaining criticism and suggestion. It is hoped that each member of the Seminary will steadily produce from time to time finished work suitable for publication. Emphasis will be placed on accurate and detailed work upon obscure or untouched points.

Students may carry on an independent study upon some special subject, making regular reports to the seminary; or, several students may be grouped for the study of a series of connected subjects. For this purpose, during 1892-3, the following topics are offered:

(a) American Shipping, with a retrospect to the experience of Great Britain and Holland since 1650, and a comparative study of modern European policies.

(b) A Study of Modern Currency Problems, treated theoretically and historically.

(c) A critical and historical examination of the Internal Revenue System of the United States.

 

FELLOWSHIPS.

Independently of the fellowships offered by the departments of Political Science and Social Science, at least three Fellowships, yielding an annual income of $500, will be assigned to students within the department of Political Economy for the year 1892-3. Appointments will be made only on the basis of marked ability in economic studies, and of capacity for investigation of a high character. Candidates for these fellowships should send to the President of the University a record of their previous work and distinctions, degrees and past courses of study, with copies of their written or printed work in economics. Applications for 1892-3 should be sent in not later than June 1, 1892.

Fellows are forbidden to give private tuition, and will be called upon for assistance in the work of teaching in the University; but in no case will they be expected or permitted to devote more than one-sixth of their time to such service.

 

PUBLICATIONS.

As a means of communication between investigators and the public, the University will issue quarterly The Journal of Political Economy, beginning in the autumn of 1892. Contributions to its pages will be welcomed from writers outside as well as inside the University, the aim being not only to give investigators a place of record for their researches, but also to further in every possible way the interests of economic study throughout the country. The Journal will aim to lay more stress than existing journals upon articles dealing with practical economic questions. The editors will welcome articles from writers of all shades of economic opinion reserving only the privilege of deciding as to merit and timeliness.

Longer investigations, translations of important books needed for American students, reprints of scarce works, and collections of materials will appear in bound volumes in a series of Economic Studies of the University of Chicago. Announcement of works already in preparation will be made at an early date.

 

LIBRARIES.

In the suite of class-rooms occupied by the department will be found the Economic Library. Its selection has been made with great care, in order to furnish not only the books needed for the work of instruction in the various courses, but especially collections of materials for the study of economic problems. It is believed that ample provision has thus been made for the work of serious research. The work of the students will necessarily be largely carried on in this Library.

Arrangements have been made with other libraries in the city for supplementing the Economic Library of the University on a large and generous scale. The combined library facilities of Chicago are exceptional. The Public Library, maintained by a large city tax; the Newberry Library, under the supervision of W. F. Poole, with a fund of several millions of dollars; and other possibilities, will enable the student to obtain any books he may need in the prosecution of detailed investigation. In the near future, it is confidently believed, the supply of reference books for students in the libraries of Chicago will be greater than anywhere else in this country; and graduate students will have exceptional opportunities for specialized research.

The officers of the department will cheerfully answer any inquiries from institutions looking for suitable teachers of Political Economy.

Inquiries and applications of students should be addressed to

THE EXAMINER,
The University of Chicago,
Chicago.

 

Source:  The University of Chicago: programme of courses in political economy, 1892-1893. Chicago, IL : University of Chicago Press, 1892.

Image Source: University of Chicago yearbook, Cap and Gown , 1900 , p. 19.

Categories
Chicago Economics Programs Economist Market Economists NYU

Chicago. Chester Wright recounts J. Laurence Laughlin to Alfred Bornmann in 1939

 

 

In 1939 a NYU graduate student, Alfred H. Bornemann, wrote to the University of Chicago economic historian Chester W. Wright requesting any of the latter’s personal memories of the first head of the Chicago Department of Political Economy, J. Laurence Laughlin. Bornemann’s letter and Wright’ response are transcribed below. Results from Bornemann’s project were published in 1940 as J. Laurence Laughlin: Chapters in the Career of an Economist. I have added Bornemann’s AEA membership data from 1948 and his New York Times obituary to round out the post.

Reading Wright’s letter it is easy to convince oneself that any oral history interview is more likely to extract something from a witness than is an open-ended request for a written statement. Still, an artifact is an artifact and Wright’s response is now entered into the digital record.

________________________________

1948 Listing in the AEA Membership Roll

BORNEMANN, Alfred H., 1618 Jefferson Ave., Brooklyn 27, N. Y. (1939). Long Island Univ., teach., res.; b. 1908; B.A., 1933, M.A., 1937, Ph.D., 1941, New York. Fields 7 [Money and Banking; Short-term Credit; Consumer Finance], 6 [Business Fluctuations].

Source:   “Alphabetical List of Members (as of June 15, 1948).” The American Economic Review 39, no. 1 (1949): 1-208. .p. 20.

________________________________

Alfred Bornemann, 82, Economist and Author
New York Times Obituary of May 3, 1991

Alfred H. Bornemann, an economist who taught at several colleges and who wrote extensively on economics, died on Friday at his home in Englewood, N.J. He was 82 years old.

He died of liver and colon cancer, his family said.

Dr. Bornemann was a professor at Norwich University and chairman of its department of economics and businness administration from 1951 to 1958. He taught at C. W. Post College of Long Island University from 1960 to 1966 and at Hunter and Kingsborough Colleges of the City University of New York from 1967 to 1974.

He wrote, among other books, “Fundamentals of Industrial Management,” published in 1963; “Essentials of Purchasing” (1974) and “Fifty Years of Ideology: A Selective Survey of Academic Economics” (1981).

Dr. Bornemann was born in Queens and received bachelor’s, master’s and doctoral degrees from New York University. He was an accountant with Cities Service and with the American Water Works and Electric Company before beginning his teaching career at N.Y.U. in 1940.

He is survived by his wife, the former Bertha Kohl; a son, Alfred R., of Bayonne, N.J., and a brother, Edwin, of Liberty, N.Y.

Source: New York Times Obituaries, May 3, 1991.

________________________________

Bornemann’s book and doctoral thesis about J. Laurence Laughlin

Alfred Bornemann. J. Laurence Laughlin: Chapters in the Career of an Economist. Introduction by Leon C. Marshall. (Washington,: American Council on Public Affairs,1940).

Chief sources: Agatha Laughlin’s recollections of her father; Letters from numerous colleagues and students; Laughlin papers in the University of Chicago and in the Library of Congress. His 300 odd books and articles published, 1876-1933.

Source: FRASER. Committee on the History of the Federal Reserve System. Biographies, Memoirs, Personal Reminiscences: American: U. Economists (Date 1956).

Downloadable doctoral thesis

Bornemann’s 1940 NYU PhD thesis (degree awarded in 1941) on J. Laurence Laughlin. 420 typewritten leaves (LOC: LD3907/.G7/1941/.B6). Downloadable pdf copy of the dissertation for libraries with access to ProQuest Dissertations & Theses Global!

________________________________

Handwritten letter from Alfred Borneman to Chester W. Wright requesting personal observations of J. L. Laughlin and the Department of Political Economy of the University of Chicago

1618 Jefferson Ave.,
Brooklyn, NY.
Jan 12, 1939.

Professor C. W. Wright,
University of Chicago,
Chicago, Illinois.

Dear Professor Wright,

I am writing a thesis on J. Laurence Laughlin, as I believe Professor Mayer has already told you. What I am trying to do, among other things, is to write a chapter on “Faculty, Fellows and Students” in Laughlin’s Department at Chicago. In this chapter, I hope to tell as much as I can about the background in the Department and about the men connected with it.

As I understand it, you were appointed instructor in 1907, assistant professor in 1910, and associate professor in 1913. Can you tell me anything of interest in connection with your original appointment, that is, where you were teaching and where you got the Ph.D.? Marshall, I think, was also appointed in 1907, but even though he did not have the Ph.D. he was made a professor in 1911. Can you suggest the reason for his more rapid advancement?

On the other hand, I may suggest that apparently you and Marshall and Field were the first to be advanced so rapidly. In any event you seem to have been advanced more rapidly than Veblen and Hoxie. It is possible that in the early days he had a different attitude.

Of course there is so much which you experience under Laughlin that would be of value to me to know about that I scarcely know how to ask you anything. Alvin Johnson has suggested that Laughlin was a neurotic and he would explain him in psychological terms, which, of course, I shall not do. But his characterization may suggest some thoughts to your mind. Moulton, incidentally, says Johnson could never have known Laughlin well enough to arrive at his conclusion, because Laughlin had few intimate friends.

I do not know, of course, how much interest you had in Laughlin’s public work or his theories, so that what I am asking you largely concerns his Department. If you care to give me any observations with respect to these two phases, however, I should naturally greatly appreciate your doing so.

But I believe you could give me most invaluable information by your recollections of your years under Laughlin and how he saw the Department, as well as possibly some of the background.

For anything which you can find the time to tell me I shall be grateful.

Cordially yours,

Alfred Borneman

 

Carbon copy of Chester W. Wright’s reply to Alfred Borneman

February 27, 1939

Mr. Alfred Borneman
1618 Jefferson Avenue
Brooklyn, New York

My dear Mr. Borneman:

I am sorry to have been so long in replying to your inquiry, but have been very rushed the last few weeks and assumed there was no need for an immediate answer.

I presume Professor Laughlin’s attention was called to me by the staff at Harvard as it seems to have been his policy to make inquiries there when he had positions to be filled. I received my Ph.D. degree at Harvard in 1906 and during the following year taught at Cornell University. It was while I was there that I received a request from Professor Laughlin to meet him for an interview in Philadelphia, following which he offered me the appointment at Chicago which I decided to accept.

Professor Marshall came to Chicago at the same time. As I recollect, he had been teaching at Ohio Wesleyan for several years after completing two or three years of graduate work at Harvard, though he did not remain there to write a thesis and get his Ph.D. degree. Since he was recognized as an excellent teacher and very competent in administrative work, the fact that he did not have a Ph.D. degree was never considered an obstacle to his promotion any more that in the case of J. A. Field, who only held a Bachelor’s degree. I presume the explanation for the more rapid advancement of the men who came to the Department at Chicago about this time is that they proved to be more of the type in whom Laughlin had confidence. President Judson, I believe, had unusual confidence in Laughlin, so the latter was able to get his recommendations approved.

Of the men already in the Department when I came, Cummings and Hill were not conspicuous successes either as teachers or productive scholars. I suspect there was no pressure either to promote them or to keep them when they had chances to go elsewhere. Just why Davenport left, I never knew. Hoxie was eventually made a full professor on the strength of his recognized success as a teacher and a student of labor problems despite views on these problems which must have seemed rather questionable to one of Laughlin’s conservatism.

Professor Laughlin was very much a gentleman of the old school and placed considerable emphasis on what he called “a sense of form.” Possibly the fact that he thought the men coming into the Department about my time and later had more of this sense of form may have been a factor in their advancement. It has never occurred to me that Laughlin was of the neurotic type, though Hoxie was.

As Laughlin’s theoretical and public work was entirely outside of my field of special interest, I cannot very profitably discuss it.

In his conduct of the Department, I had no feeling that he was autocratic or unreasonable. My recollection is that most matters of general interest were discussed among the members of the Department and commonly acted upon as decided by the group. I suspect that this may have been more generally the case after about the time I came to the Department here than it had been formerly, but I have no definite knowledge on this point.

Sincerely yours,

Chester W. Wright

CWW-W

Source: University of Chicago Archives. Department of Economics, Records. Box 41, Folder 12.

Image Source:  Dr. Alfred Bornemann in C. W. Post College Yearbook, 1966.

Categories
Amherst Chicago Columbia Economists

Columbia. John Maurice Clark. Autobiographical notes, 1949

 

The following recollections of John Maurice Clark of his earliest contacts with economic problems is found in a folder of his papers containing notes about his father, John Bates Clark. The hand-written notes are fairly clear until we come to a clear addition on the final page. Abbreviations are used there and the handwriting is not always clear. Still the pages together provide a few nice stories and short lists of J.M. Clark’s teachers and students.

______________________

June 8, 1949

J.M.C.’s recollections of his earliest contacts with economic problems.

I think my earliest contact with an economic problem came on learning that the carpenter who sometimes came to do odd jobs for us at 23 Round Hill got $2.00 a day. I had a special interest in that carpenter. He was a tall man, with a full, dark beard; and it had been my imprudent interest in his operation with the kitchen double-windows (putting on? taking off?) that led me to lean out of a hammock and over the low rail of our second-story porch, to watch him (I was between two and three at the time). Mechanical consequences—I descended rapidly, landing on my head, but apparently suffering no injury except biting my tongue. Subjective consequences – maybe it pounded a little caution into me at an early age; but the present point is that it fixed that carpenter in my memory as “the man who picked me up.” It was some time later I learned that he got $2.00 a day.

I don’t remember whether I took the initiative and asked, or not. The cost of things was often discussed in our house, and my mother often talked of the difficulty of making both ends meet. I knew my father’s salary, though I can’t be sure now whether it was $3,500 or less. Anyhow, it was maybe eight or ten times the carpenter’s pay; and I began wondering how he made both ends meet, and remarked to my father that $2.00 a day wasn’t much to live on. He answered that it was pretty good pay for that kind of work. So I learned there were two ways of looking at a daily stipend—as income to live on and as the price of the service you gave your employer. Or perhaps simply the standpoints of the recipient and the payer. But especially I learned there were people who had to adjust their ideas of what they could live on, to a fraction of the income we found skimpy for the things we thought of as necessary. In short, I had a lesson in classes and their multiple standards to ponder over; without reaching any very enlightening conclusions.

I don’t think I connected this with our friends the Willistons (of the family connected with Williston seminary in Easthampton) who lived in the big house above us and from whom we rented ours. They were evidently much richer than we. They had gone to Europe (and been shipwrecked on the way, and had to transfer at sea to a lumber-schooner, which threw its deckload of lumber overboard to enable it to take on the people from the helpless steamship. — but that’s another story.)

To return to the carpenter. I suppose today he’d get perhaps $16, more?, and a Smith College salary, for a full professor, might be $7,000 or $8,000. The discrepancy has shrunk to maybe 2/5—certainly less than half—of what it was then. That puzzling discrepancy was my first lesson in economics—the first I remember.

There was another lesson—if you could call it that—the summer we spent a while at the Stanley House (now gone) in Southwest Harbor, on Mt. Desert. The rich people went to Bar Harbor. At Southwest, there was Mr. Brierly who had a yacht. We took our outings in a rowboat, sometimes with the help of a spritsail. One time we were going up Somes Sound, and were passed by one of the biggest ocean-going steam yachts—the “Sultana”. It was a very impressive sight, in those narrow waters, and looked about as big as the “Queen Mary” would to me now. I don’t remember anybody doing any moralizing; but if they did, the impression it left was that we, in our fashion, were doing the same kind of thing they were.

My first contact with economic literature (not counting the subversive economics of Robin Hood, which we boys knew by heart, in the Howard Pyle version) was at 23 Round Hill, so I must have been less than nine. I found a little book on my father’s shelves that had pictures in it – queer pictures done in pen and ink, which puzzled me. There was a boy not much bigger than I was, in queer little knee-britches, acting as a teacher to a class of grown men (including I think a Professor Laughlin, under whom I later taught at the University of Chicago.) And there were classical females being maltreated by brutal men, and other queer things. I was curious enough to read some of the text, to find out about the pictures. It was “Coin’s Financial School,” the famous free-silver tract.

I read enough to become a convinced free-silverite. And then I had the shock of discovering that my beloved and respected father was on the wrong side of that question. I decided there must be more to it than I’d gotten out of the queer picture-book. I suppose that was my first lesson in the need of preserving an open mind and holding economic ideas subject to possible reconsideration. Davenport and Veblen gave me more extensive lessons, fifteen or twenty years later, only this second time it was my father’s ideas I had to rethink, after reluctantly admitting that these opposing ideas represented something real, that needed to be reckoned with. One had to do something about it, though the something didn’t mean substituting Veblen for my father. It was a more difficult and discriminating adjustment that was called for.

To return to my boyhood. It may have been about this time that I learned something about mechanical techniques, when my father took me to see the Springfield Arsenal. They had a museum, with broadswords that had been used in battle—one was so nicked up that its edge had disappeared in a continuous series of surprisingly deep nicks—but the mechanical process that impressed me was a pattern-lathe, rough-shaping the stocks of Krags. On one side was a metal model of the finished stock revolving, with a wheel revolving against it. On the other side was the wooden blank revolving, and a wheel like the one on the model, and linked to it so as to copy its movements, and armed with knives. So the machine could make complicated shapes following any model you put into it, and do it faster and more accurately that a hand worker.

Incidentally (and as a digression) that was our first military rifle with smokeless powder, more powerful than black; our first regular military magazine rifle of the modern kind with a bolt action and a box magazine. The regulars were just getting them. The militia still had the black-powder 45-70 Springfields at the time of the Spanish War, and a Massachusetts regiment had to be ordered off the firing-line at El Caney because their smoke made too good a target. Teddy Roosevelt had pull enough to get Krag carbines for his Rough Riders plus the privilege of using their own Winchesters if individuals preferred, and, if they had the 30-40-220, which took the Krag cartridge.

But my regular education in economic theory began at the age of 9 or 10, in our first year at Amherst, when we lived on Amity Street, opposite Sunset Ave. My father had in mind James Mill’s training of his son, John Stuart Mill, and he copied the techniques of explaining something during a walk, but he didn’t follow James Mill’s example by making me submit a written report for criticism and revision. All he did was to explain about diminishing utility and marginal utility—using the illustration of the oranges. And he was satisfied that I understood it, and concluded that the simple fundamentals of economics could be taught to secondary school or “grammar-school” students. Later, my friend and former graduate student, Leverett Lyon, pithily remarked that I probably understood it better then than I ever had since. Maybe he was right. I know when I met Professor Fetter, the year the Ec. Ass. met in Princeton, he told me I didn’t understand the theory, because I had said (in print, I think) that there were some dangers about the concept of “psychic income.” I didn’t say it was wrong, but I did think it was likely to be misleading to use a term that was associated with accountants’ arithmetic. So I did probably understand the theory “better” at the age of 9 or 10. Twenty ears later, it didn’t look so simple. This was long before I disagreed with Fetter about basing-point pricing and the rightness of the uniform FOB mill price, as the price “true” competition would bring about.

______________________

J.M.C. later history.

Amherst, C in Ec tho 85 on exam, & written work not credited. (cf French A from Wilkins, C from [William Stuart] Symington (father of present (1951) W. Stuart Symington, head of nat security Resources Board). Symie sized my attitude up as that of a gentleman & gave me a gentleman’s mark)ache Crook said he “didn’t get hold” of me. He was correct.

 

Columbia: Giddings, A. S. Johnson, H.L. Moore, Seligman, Seager, Hawkins [?], Chaddock, Agger, Jacobstein. indoctrinated: J. B. C. orthodoxy modified by overhead costs (catalogued as “dynamics”) Dynamics (defined as) everything statics leaves out. & much induction. Take “Essentials” on slow dictation.

Veblen: slow infiltration of its logical & progre[?] rel. to the abstractions of J.B.C.: reverse normalizing might make[?] an arguable claim to equal legitimacy.

1912 ed. of Control of Trusts

“Contribution to theory of competive price” [QJE, August 1914] forerunner of “mon-comp”, largely empirical basis.

Germs of social & inst. ec. Rich-poor, Freedom as val in ec.[??] B. M. Anderson cf. Cooley

Revs of Hobson?, Pigou, Davenport Economics of Enterprise [Political Science Quarterly, Vol 29, no. 2]

 

To Chi. 1915 Changing basis of economic responsibility [JPE, March 1916] on moving to Chi. open declar[ation] of non-Laughlinism: backfire to an Atlantic article of Laughlin’s.

Modern Psych.

1917-18. War-ec. (“basis of war-time collectivism.”)

Students: Garver oral. Slichter, Lyon, Innis, Martin [?], Goodrich, Copeland, O’Grady [John O’Grady ?]

Ayres, Knight on faculty.

Ov. C. [Studies in the Economics of Overhead Costs]

Social Control [of Business]

 

Columbia. Students, Friedman, Ginzberg, Salera, Kuznets’ oral

 

Source: Columbia University Archives. John M. Clark Collection. History of Economic Thought. Box 37, Folder “J. B. Clark, 1847-1938”.

Image Source: John Maurcie Clark. University of Chicago Photographic Archive, apf1-0171.  Special Collections Research Center, University of Chicago Library.

Categories
Berkeley Chicago Columbia Economists NBER New School

Columbia. Memorial Minute for Wesley Clair Mitchell, 1949

 

Memorial minutes entered into a faculty’s record have the virtue of being brief and typically are written by someone who has had a close personal/professional relationship with the subject as seen in the following memorial minute delivered by Wesley Clair Mitchell’s student and later colleague, Frederick C. Mills.

The dual memoir Two Lives–The Story of Wesley Clair Mitchell and Myself, written by Mitchell’s wife Lucy Sprague Mitchell is available at hathitrust.org and provides much detail, e.g. an eight page autobiographical letter written by Mitchell in 1911.

______________________

WESLEY CLAIR MITCHELL
Memorial Minute read by Professor F. C. Mills
February 18, 1949

Wesley Clair Mitchell, Professor Emeritus of Economics, died in New York City on October 29, 1948. In his death the world lost one of the great scholars of our generation and the members of this Faculty lost a distinguished colleague and a cherished friend.

Wesley Mitchell was born in Rushville, Illinois, on August 5, 1874, the son of a country doctor who had won the rank of Brevet Colonel as a Civil War surgeon. The family was of New England stock, and although a middle-western boyhood and later adult years in California and New York left their impress on Mitchell, something of the New England strain was always discernible in the pattern of his thought and life.

Mitchell’s student days, undergraduate and graduate, were spent at the University of Chicago, with a one-year interim period at Halle and Vienna. The influence of the German and Austrian residence was slight; Mitchell was a product of American university training in the period of vigorous growth that came at the turn of the century. His outstanding qualities as an economist were distinctive of ways of thought and study that were largely indigenous to this country. Thorstein Veblen, John Dewey, J. Laurence Laughlin in their several ways deeply affected Mitchell’s thinking and his way of conceiving of the problems of society.

Following a year at the Census Bureau and a short term as instructor at the University of Chicago, Mitchell moved in 1902 to the University of California, at Berkeley, to begin a decade of fruitful work and of steady personal growth. His tools of research were sharpened and his mastery of them perfected. The brilliant studies of the greenback period, in which the pattern of his scholarly work was first defined, were extended. The massive monograph on Business Cycles, one of the great products of scholarship in the social sciences, was here completed. But beyond these solid contributions to economic thought and method this was a rich period inMitchell’s life, to which he always looked back as something of a personal golden age. A young man intellectually somewhat aloof and inclined toward austerity mellowed in the sunshine of the west and in the easy, pleasant companionships of the young University. He took to the Sierras avidly, relishing the free ways, the free language and the physical release to be found in mountain climbing. A companion of those days says that Wesley’s inhibitions were peeled off like the layers of an onion as successive altitude levels were passed. He found a wife, too, in the west; when he left California in 1912 he took with him the Dean of Women of the University.

Wesley Mitchell’s service at Columbia began in 1913 and extended to the date of his retirement in 1944, except for a three-year term at the New School for Social Research. Indeed, his Columbia connection extended, properly, to the day of his death, for there was no time when we did not consider him one of us, or when he did not so regard himself. Mitchell’s reputation had been established by the time he came to Columbia; he had reached full scholarly maturity. Yet his growth continued and his accomplishments multiplied. A steady (but not a voluminous) flow of papers, reviews, addresses and more extensive studies came from his pen. Into each, whether brief or extended, went care in the construction of a logical and orderly argument, skill in the marshaling of evidence, and objectivity in the use of that evidence. Each, too, was in exposition a work of craftsmanship by a man whose ear was extraordinarily sensitive to the rhythms of our language and whose mind was alert to shades of meaning and subtleties of expression.

There was also an almost uninterrupted series of public and professional services and of accumulating honors. He was Chief of the Price Section of the War Industries Board during the first World War, chairman of the President’s Committee on Recent Social Trends, a member of the National Planning Board, the National Resources Board, and the Federal Emergency Administration of Public Works, and chairman of the Committee on the Cost of Living when that burning issue threatened to check the steady production of goods during the second World War. There was the launching in 1920 and the directing for a quarter of a century of a new instrument for the advancement of knowledge—the National Bureau of Economic Research. Over a long stretch of years he helped to break down the barriers between the social sciences and to unify their activities in the Social Science Research Council. He was one of those who founded and shaped the New School for Social Research. Counsel and guidance were given over many years to the Bureau of Educational Experiments. He was called upon to direct the affairs of professional societies, serving as President of the American Economic Association, the American Statistical Association, the Econometric Society, and the American Association for the Advancement of Science. There were elections to learned societies at home and abroad. Honorary degrees came from Oxford, the University of Paris, and from major universities in this country. These were rich honors and they were not unwelcome; but he remained to the day of his death a modest scholar, who would both gladly learn and gladly teach.

It was as teacher and scholar that Mitchell’s greatest services were rendered to Columbia, and it was in these roles that he was best known to us of this Faculty. Mitchell possessed in high degree the qualities of a good teacher. There was insight in his analyses; there was a freshness of view that he never lost; there was lucidity of thought and expression; there was a sense of sharing with the student the task of inquiry. Above all, perhaps, was the sense of integrity. Here was a man without affectation, without pretense, who honestly sought understanding.

The specific contributions that Mitchell made to economics will be duly appraised by his colleagues in that profession. As members of a political science faculty, however, it is proper for us to recognize the service of Mitchell in breaking economics out of the tight formalism of the tradition that prevailed when he came to the subject. He was profoundly unhappy about economics as a branch of logic, dealing with the interaction of atoms in the form of human reasoning machines, subjecting itself only to tests of logical consistency, almost indifferent to the relevance of its principles to complex and constantly changing reality. Mitchell himself was not unskilled in the spinning of deductive arguments, but he was keenly aware of the dangers of self-delusion in unchecked rationalism. His bent was empirical; his emphasis in research was on the constant checking of reason against observation. First in the monetary field, later in the study of prices, of business cycles, and of national income, he developed and refined methods of quantitative analysis and stimulated a movement that has deeply affected the character of economic research and the content of economic thought the world over. But Mitchell’s concern was never with method as method. Man was at the center. Economics was to him on of the sciences of human behavior. And the human being with whose actions he was concerned was a complex creature whose motives could not be reduced to the reasoned balancing of satisfactions against pains or of prospective gains against prospective losses. He stressed the role in economics of institutions — of money, of the industrial system — which man had shaped and which in turn were shaping him; in so doing he helped to turn many younger economists to the study of a neglected phase of economic life. These various aspects of Mitchell’s thought are developed in treatises and shorter papers published over a period of fifty years. They are outstandingly revealed in the series of books on business cycles that are Mitchell’s greatest substantive contribution to economics.

Some of the personal qualities of Wesley Mitchell have been suggested in this brief account of his work. But there was much more than this. He was a lover of poetry whose mind was stocked with verse. He was a connoisseur of mystery stories who could warmly resent the moral betrayal of the reader when the author played unfairly with him. He was a craftsman, skilled in the fine art of woodwork. He was tenacious and unremitting in seeking principles of order in human affairs, yet free from dogmatism and open to criticism and advice from his youngest associates. He was a kindly and generous man, a source of continuing and friendly inspiration to students and colleagues alike. In his life’s work Mitchell served the human race. In his own being he helped to give dignity to that race.

 

Source: Memorial Minute on Professor Wesley C. Mitchell read by Professor F. C. Mills at the meeting of Faculty of Political Science of February 18, 1949. Appended to the Minutes of the Faculty Meeting.

Image Source:Foundation for the Study of Cycles Website  .

Categories
Chicago Economists Funny Business

Chicago. Economics Ph.D. (1903), Canadian Humorist Stephen Leacock.

It is not every day that one stumbles upon a history-of-economics arc connecting Thorstein Veblen to Groucho Marx and Jack Benny. The economist that connected the iconoclast economist to those veterans of vaudeville comedy is the Canadian humorist and Chicago student of Thorstein Veblen, Stephen Butler Leacock.

First I post here some data (the actual starting point of my background check of Leacock, the Chicago Ph.D.) found in the University of Chicago’s registers of its Ph.D.’s and annual catalogues.

The author’s autobiographical Preface to Leacock’s greatest hit, Sunshine Sketches of a Little Town (1912) follows. The stories themselves strike most, if not all, of the same chords that Garrison Keillor’s News from Lake Wobegon has played over the past decades. 

Finally I will allow myself the short-cut of quoting Wikipedia to complete the sketch of both sides of this most interesting fellow. 

The McGill economics department entry for Stephen Leacock.

 

_____________________

Stephen Butler Leacock
University of Chicago Ph.D. in Political Economy, 1903.

Thesis Title: The doctrine of laissez faire.

 A.B. University of Toronto, 1891.

1889-99. Instructor in French and German, Upper Canada College.
1899-1900. Graduate Student, University of Chicago.
1921. Head of Department of Economics and Political Science, McGill University, Montreal, Canada.
1931, April 1. Professor and Head of Department of Economics and Political Science, McGill University, Montreal, Canada.
1938, April 1. Professor Emeritus of Economics and Political Science, McGill University, Montreal, Canada.

 

_____________________

Author’s Preface to Sunshine Sketches of a Little Town (1912)

               I KNOW no way in which a writer may more fittingly introduce his work to the public than by giving a brief account of who and what he is. By this means some of the blame for what he has done is very properly shifted to the extenuating circumstances of his life.

I was born at Swanmoor, Hants, England, on December 30, 1869. I am not aware that there was any particular conjunction of the planets at the time, but should think it extremely likely. My parents migrated to Canada in 1876, and I decided to go with them. My father took up a farm near Lake Simcoe, in Ontario. This was during the hard times of Canadian farming, and my father was just able by great diligence to pay the hired men and, in years of plenty, to raise enough grain to have seed for the next year’s crop without buying any. By this process my brothers and I were inevitably driven off the land, and have become professors, business men, and engineers, instead of being able to grow up as farm labourers. Yet I saw enough of farming to speak exuberantly in political addresses of the joy of early rising and the deep sleep, both of body and intellect, that is induced by honest manual toil.

I was educated at Upper Canada College, Toronto, of which I was head boy in 1887. From there I went to the University of Toronto, where I graduated in 1891. At the University I spent my entire time in the acquisition of languages, living, dead, and half- dead, and knew nothing of the outside world. In this diligent pursuit of words I spent about sixteen hours of each day. Very soon after graduation I had forgotten the languages, and found myself intellectually bankrupt. In other words I was what is called a distinguished graduate, and, as such, I took to school teaching as the only trade I could find that needed neither experience nor intellect. I spent my time from 1891 to 1899 on the staff of Upper Canada College, an experience which has left me with a profound sympathy for the many gifted and brilliant men who are compelled to spend their lives in the most dreary, the most thankless, and the worst paid profession in the world. I have noted that of my pupils, those who seemed the laziest and the least enamoured of books are now rising to eminence at the bar, in business, and in public life; the really promising boys who took all the prizes are now able with difficulty to earn the wages of a clerk in a summer hotel or a deck hand on a canal boat.

In 1899 I gave up school teaching in disgust, borrowed enough money to live upon for a few months, and went to the University of Chicago to study economics and political science. I was soon appointed to a Fellowship in political economy, and by means of this and some temporary employment by McGill University, I survived until I took the degree of Doctor of Philosophy in 1903. The meaning of this degree is that the recipient of instruction is examined for the last time in his life, and is pronounced completely full. After this, no new ideas can be imparted to him.

From this time, and since my marriage, which had occurred at this period, I have belonged to the staff of McGill University, first as lecturer in Political Science, and later as head of the department of Economics and Political Science. As this position is one of the prizes of my profession, I am able to regard myself as singularly fortunate. The emolument is so high as to place me distinctly above the policemen, postmen, street-car conductors, and other salaried officials of the neighbourhood, while I am able to mix with the poorer of the business men of the city on terms of something like equality. In point of leisure, I enjoy more in the four corners of a single year than a business man knows in his whole life. I thus have what the business man can never enjoy, an ability to think, and, what is still better, to stop thinking altogether for months at a time.

I have written a number of things in connection with my college life — a book on Political Science, and many essays, magazine articles, and so on. I belong to the Political Science Association of America, to the Royal Colonial Institute, and to the Church of England. These things, surely, are a proof of respectability. I have had some small connection with politics and public life. A few years ago I went all round the British Empire delivering addresses on Imperial organization. When I state that these lectures were followed almost immediately by the Union of South Africa, the Banana Riots in Trinidad, and the Turco-Italian war, I think the reader can form some idea of their importance. In Canada I belong to the Conservative party, but as yet I have failed entirely in Canadian politics, never having received a contract to build a bridge, or make a wharf, nor to construct even the smallest section of the Transcontinental Railway. This, however, is a form of national ingratitude to which one becomes accustomed in this Dominion.

Apart from my college work, I have written two books, one called “Literary Lapses” and the other “Nonsense Novels.” Each of these is published by John Lane (London and New York), and either of them can be obtained, absurd though it sounds, for the mere sum of three shillings and sixpence. Any reader of this preface, for example, ridiculous though it appears, could walk into a bookstore and buy both of these books for seven shillings. Yet these works are of so humorous a character that for many years it was found impossible to print them. The compositors fell back from their task suffocated with laughter and gasping for air. Nothing but the invention of the linotype machine or rather, of the kind of men who operate it made it possible to print these books. Even now people have to be very careful in circulating them, and the books should never be put into the hands of persons not in robust health.

Many of my friends are under the impression that I write these humorous nothings in idle moments when the wearied brain is unable to perform the serious labours of the economist. My own experience is exactly the other way. The writing of solid, instructive stuff fortified by facts and figures is easy enough. There is no trouble in writing a scientific treatise on the folk-lore of Central China, or a statistical enquiry into the declining population of Prince Edward Island. But to write something out of one’s own mind, worth reading for its own sake, is an arduous contrivance only to be achieved in fortunate moments, few and far between. Personally, I would sooner have written “Alice in Wonderland ” than the whole Encyclopaedia Britannica.

In regard to the present work I must disclaim at once all intention of trying to do anything so ridiculously easy as writing about a real place and real people. Mariposa is not a real town. On the contrary, it is about seventy or eighty of them. You may find them all the way from Lake Superior to the sea, with the same square streets and the same maple trees and the same churches and hotels, and everywhere the sunshine of the land of hope.

Similarly, the Reverend Mr. Drone is not one person, but about eight or ten. To make him I clapped the gaiters of one ecclesiastic round the legs of another, added the sermons of a third and the character of a fourth, and so let him start on his way in the book to pick up such individual attributes as he might find for himself. Mullins and Bagshaw and Judge Pepperleigh and the rest are, it is true, personal friends of mine. But I have known them in such a variety of forms, with such alternations of tall and short, dark and fair, that, individually, I should have much ado to know them. Mr. Pupkin is found whenever a Canadian bank opens a branch in a county town and needs a teller. As for Mr. Smith, with his two hundred and eighty pounds, his hoarse voice, his loud check suit, his diamonds, the roughness of his address and the goodness of his heart, all of this is known by everybody to be a necessary and universal adjunct of the hotel business.

The inspiration of the book, —a land of hope and sunshine where little towns spread their square streets and their trim maple trees beside placid lakes almost within echo of the primeval forest, is large enough. If it fails in its portrayal of the scenes and the country that it depicts the fault lies rather with an art that is deficient than in an affection that is wanting.

STEPHEN LEACOCK.

McGill University,
June, 1912.

Source: Stephen Leacock, Sunshine Sketches of a Little Town, London: John Lane, 1912, pp. vii-xii.

 

_______________________

Academic and political life

Disillusioned with teaching, in 1899 he began graduate studies at the University of Chicago under Thorstein Veblen, where he received a doctorate in political science and political economy. He moved from Chicago, Illinois to Montreal, Quebec, where he eventually became the William Dow Professor of Political Economy and long-time chair of the Department of Economics and Political Science at McGill University.

He was closely associated with Sir Arthur Currie, former commander of the Canadian Corps in the Great War and principal of McGill from 1919 until his death in 1933. In fact, Currie had been a student observing Leacock’s practice teaching in Strathroy in 1888. In 1936, Leacock was forcibly retired by the McGill Board of Governors—an unlikely prospect had Currie lived.

Leacock was both a social conservative and a partisan Conservative. He opposed giving women the right to vote, disliked non-Anglo-Saxon immigration and supported the introduction of social welfare legislation. He was a staunch champion of the British Empire and the Imperial Federation Movement and went on lecture tours to further the cause.

Although he was considered as a candidate for Dominion elections by his party, it declined to invite the author, lecturer, and maverick to stand for election. Nevertheless, he would stump for local candidates at his summer home.

Literary Life

Early in his career, Leacock turned to fiction, humour, and short reports to supplement (and ultimately exceed) his regular income. His stories, first published in magazines in Canada and the United States and later in novel form, became extremely popular around the world. It was said in 1911 that more people had heard of Stephen Leacock than had heard of Canada. Also, between the years 1915 and 1925, Leacock was the most popular humorist in the English-speaking world.

A humorist particularly admired by Leacock was Robert Benchley from New York. Leacock opened correspondence with Benchley, encouraging him in his work and importuning him to compile his work into a book. Benchley did so in 1922, and acknowledged the nagging from north of the border.

Near the end of his life, the American comedian Jack Benny recounted how he had been introduced to Leacock’s writing by Groucho Marx when they were both young vaudeville comedians. Benny acknowledged Leacock’s influence and, fifty years after first reading him, still considered Leacock one of his favorite comic writers. He was puzzled as to why Leacock’s work was no longer well known in the United States.

During the summer months, Leacock lived at Old Brewery Bay, his summer estate in Orillia, across Lake Simcoe from where he was raised and also bordering Lake Couchiching. A working farm, Old Brewery Bay is now a museum and National Historic Site of Canada. Gossip provided by the local barber, Jefferson Short, provided Leacock with the material which would become Sunshine Sketches of a Little Town (1912), set in the thinly-disguised Mariposa.

Although he wrote learned articles and books related to his field of study, his political theory is now all but forgotten. Leacock was awarded the Royal Society of Canada’s Lorne Pierce Medal in 1937, nominally for his academic work.

Source: From the Wikipedia article “Stephen Leacock”.

Image Source: PMA Productions, Extraordinary Canadians. Margaret Macmillan’s Episode on Stephen Leacock.

Categories
Business School Chicago Curriculum

Chicago. Laughlin on Establishing a Business School, 1895

Basic training in graduate education in economics has been distilled into a trinity of microeconomics, macroeconomics and econometrics. This tends to be taken for granted by most economics departments. However long before we ever got here, “political economy” or “economics” has coexisted with history, business, sociology and public affairs, perhaps each within a separate cubicle but all nevertheless sharing a common office space. We see in today’s posting for the University of Chicago that the branching off of business studies occurred fairly early in the development of U.S. graduate/professional education.

I think this sort of development is important to follow because once administrative walls have been built, interdisciplinarity gets reduced to Pyramus and Thisbe interactions. (Plot spoiler: it didn’t end well for that couple.)

The following interview with the head of the Chicago department of political economy, J. Laurence Laughlin, provides us with an ex ante view of business education.

________________________________________

 

NO SCHOOL IS LIKE IT
SCHEME OF INSTRUCTION WITHOUT
AN AMERICAN PARALLEL.

Chicago Daily Tribune, May 12, 1895

University of Chicago’s Department of Business Economics and Journalism to Cover Wide Range of Practical Every-Day Training—Forecast of the Leading Courses—Railways to Receive Special Attention—Number of Instructors Required in the School of Economics.

“Is the University of Chicago to have a department of business and economics and journalism similar to the Wharton School of the University of Pennsylvania?” was asked Prof. J. Laurence Laughlin, head of the department of economics in the university yesterday afternoon.

“No, it is not,” he replied, “we are to have a school in business economics and journalism, but it will not be modeled after the Wharton school.”

“It seems strange,” Prof. Laughlin continued, “but the statement made by President Harper at the April convocation regarding the establishment in the university of courses in banking, transportation, insurance, consular and diplomatic service, and corporation management seems to have been entirely buried in the public mind. As a matter of fact, Dr. Harper gave utterance to a scheme the like of which has never been attempted in this country. People are familiar with schools of law, medicine, and dentistry, but the idea that a journalist, a banker, a railroad man, a diplomat, or a manager of a corporation should have special training in their particular line of employment is not readily conceived. The new work which the University of Chicago expects to undertake will, as I say, constitute a new departure in modern education. The Wharton school has an endowment of only about $100,000; the University of Chicago expects to organize its departments of business economics with no less than $1,000,000. True, these various departments of practical economic work will not deal with the arithmetic of banking or the technique of railroading or journalism. These things must be learned by practical contact with men and affairs. It is, however, necessary that a banker should be thoroughly acquainted with the principles and functions of money, that he should understand the industrial economics of his own and other countries, and that he should understand the character and extent of the changes in this own business which may be brought about by constantly arising changes in industrial economics, money legislations, etc.”

No Fear as to Results.

            “Are you not met with the objection that the training of young men to be bank Presidents, railway magnates, diplomats, etc., is in the face of present-day competition and business shifts, a rather dangerous undertaking? was asked.

“No, I do not think so. In 1880, for instance, one-fifth of those engaged in gainful pursuits in the Unite States were engaged in transportation. The business of a banker, a railroad manager, an actuary, or an expert accountant is becoming sufficiently extensive and of sufficient importance not only to warrant such training, but to make it necessary to the successful management of any one of these businesses. The fact should be emphasized that we shall not attempt the clerical part of an education in any of these lines of work. In the school of journalism we shall be satisfied if the student learns to think clearly and independently upon economic subjects and is fairly well grounded in the kind of history, law, and economics acquaintance with which every public teacher requires.”

Prof. Laughlin is Chairman of a committee, the other members of which he is not prepared to announce, which is at work upon the courses which will enter into the new curriculum. It is not known at just what time the scheme will be announced in detail, but there is no doubt that the plan will, in due time, be operated along the lines indicated. When asked whom the university would probably invite to captain the various departments of the new school Prof. Laughlin said he had nothing for publication.

The leading courses under the new scheme will undoubtedly be banking and railroading. Of the first course Prof. Laughlin will probably have charge. The course will probably deal with the comparative banking systems of the United States, England, France, Germany, Switzerland, and other countries, and special attention will be given to the manner in which each meets the problems of currency (coin, note, and deposit), reserves, discount, and exchange. The relations of the banks to the public, their influence on speculation, their management in financial crises, special dangers, and most efficient safeguards will be discussed; also relative advantages and different fields of action for national banks, State banks, deposit and trust companies, and savings banks.

Course in Railway Transportation.

            Prof. von Holst and Prof. Laughlin are thoroughly alive to the field which the railway is opening up to the student and business-man. Prof. von Holst says a competent history of the United States cannot be written until the growth and mechanism of the railway has been set forth. The course in railway transportation for the winter of 1896 suggests the character and extent of work which the new economic training will offer. The course will begin with a discussion of the economic, financial, and social influences arising from the growth of modern railway transportation, especially as concerns the United States. Then will follow an account of the means of transportation developed in Europe and America during the early part of this century, the experiments of the States in constructing and operating canals and railways; national, State, and municipal aid to private companies; the rapid and irregular extension of the Untied States railway system in recent years, with some attention to railway building in other countries.

A discussion of various theories of rates; competition, combination, discrimination, investments, speculation, abuse of fiduciary powers; State legislation and commissions and the inter-State commerce act, with decisions under it; also the various relations of the State, the public, investors, managers, and employés will form the most important part of the work. A comparison of the United States railway system with those of other countries will be made, with special attention to the problems of State ownership.

Prof. E. R. L. Gould, the statistician-elect from Johns Hopkins University, will likely be the statistician of the new school. Prof. Gould will assume his duties at the university next October. In his department Prof. Gould will trace the historical development of statistics and examine into the work of private statistical associations and of official agencies in all the leading countries. The student will be given the claims of statistics to scientific recognition, the principles of statistical judgments, and the problems of systematic statistics. Together with the necessity of uniformity of method and comparability of data, graphical methods, and cartography, attention will be drawn to the technique of statistics.

Thorough Analysis of Statistics.

            Demonstrations with actual statistical material being the most satisfactory method of statistical instruction, particular stress will be laid upon this feature of the course. Statistical returns of various sorts will be carefully analyzed and generalizations made when possible. International comparisons will also receive special attention and exposition and practical analysis will be applied in the following classes of statistics: Population, education, vital statistics, paupers, criminals and defectives, social statistics of cities, industry and labor, land and agriculture, transportation, trade and commerce, prices and public finance.

Prof. A.C. Miller will have charge of the department of finance. In this course it is intended to make a comprehensive survey of the whole field of public finance. Review will be made of the growth of and present state of the expenditures of leading modern nations, and the methods used for defraying them. Taxation, holding the place of first-importance among the resources of the modern state, will be the principal subject of the course. A critical estimate will be made of the theories of leading writers with a view to discovering a tenable basis for taxation. Special attention will be given to the comparative study of the tax systems of the principal modern states, and to the problems of State and local taxation in America. All questions will be discussed from the two-fold standpoint of justice and expediency.

The remaining pars of the course will treat of the organization and methods of financial administration, the formal control of public expenditures by means of the budget, the growth of public debts and their economic and social effect. The various problems involved in the management of public debts, such as methods of borrowing, conversion, and reduction, will be considered, and the methods practiced in our own and other countries described.

A course to be given by Dr. Thorstein B. Veblen in “Problems in American Agriculture” will be a feature of the economic work for 1895-’96. In this department special attention will be given to the extension and changes of the cultivated area of the United States; the methods of farming; the influence of railways and population and of cheapened transportation; the fall in values of Eastern farm lands; movements of prices of agricultural products; European markets; competition of other countries; intensive farming; diminishing returns; farm mortgages; and the comparison of American with European systems of culture. Systems of holdings in Great Britain, Belgium, France, and Germany will be touched upon, together with the discussion of forestry legislation.

Twenty-nine Instructors Required.

            This description of a few courses in economics announced for 1895-’96 will give some idea of the scope of work with which the new school of economics will deal. Seven instructors are registered in the department of political economy, four in political science, nine in history, and nine in sociology and anthropology—all related sciences, and each of which will probably be represented in the new school, or rather in the extension of the present school.

Besides courses in banking, railway transportation, insurance, and corporation management the new school will include courses in the consular and diplomatic service, trading and shipping, and municipal government. No attempt will be made to go into the details of these departments further than is essential to a comprehension of the mechanism and principles of the entire business.

The problem to which the University of Chicago addresses itself is the proper arrangement of the courses, the engagement of expert instructors, and the establishment of libraries and bureaus of information for the use of students.

Chicago being the greatest railway center of the United States and the home of several prominent railway managers, it is thought that certain Chicago men will be solicited for a portion of their time to be spent in university instructions, the aim being to united with a theoretical education a practical business training, unencumbered, however, with the clerical routine and forms.

“After all,” says Prof. Laughlin, “our graduated banker must begin at the bottom and work his way up like other individuals; but he will, nevertheless, have the indisputable advantage over his rival of seeing and conceiving different departments of the bank in connection with the whole. Details are, after all, easily learned. The new department will savor little of the ‘school,’ will be practical and up to date in its methods, and will give the would-be banker or railway manager or superintendent the same preparation as I now given the intending lawyer or physician in a law or medical school.”

Image Source: University of Chicago Photographic Archive, apf1-03687, Special Collections Research Center, University of Chicago Library.

Categories
Berkeley Chicago Columbia Economists

Columbia. Wesley C. Mitchell’s Methodological Thoughts, 1928.

The following excerpts from a typed copy of a letter from Wesley C. Mitchell to John Maurice Clark dated August 9, 1928 come from Mitchell’s papers with a hand-written note at the top of the first page, “Revised Feb 11, 1929”. The copy was made by Clark and perhaps given to Mitchell for further comment.

Mitchell begins with a longish response to a question posed by Clark regarding Mitchell’s own professional revealed preference for empirical investigation. This is followed by shorter responses to questions about the origin of his interest in business cycles, the relationship of “analytical description” to “causal theory”, and finally Mitchell’s confessed own perceived shortcomings in the use of statistical techniques for trend and seasonal analysis.

_____________________________________

If you find this posting interesting, here is the complete list of “artifacts” from the history of economics I have assembled. You can subscribe to Economics in the Rear-View Mirror below. There is also an opportunity for comment following each posting….

_____________________________________

Letter from Wesley C. Mitchell to John Maurice Clark
9 August 1928 (excerpt)

[…]

            Concerning the inclination you note to prefer concrete problems and methods to abstract ones, my hypothesis is that it got started, perhaps manifested itself would be more accurate, in childish theological discussions with my grand aunt. She was the best of Baptists, and knew exactly how the Lord had planned the world. God is love; he planned salvation; he ordained immersion; his immutable word left no doubt about the inevitable fate of those who did not walk in the path he had marked. Hell is no stain upon his honor, no inconsistency with love.—I adored the logic and thought my grand aunt flinched unworthily when she expressed hopes that some back-stairs method might be found of saving from everlasting flame the ninety and nine who are not properly baptized. But I also read the Bible and began to cherish private opinions about the character of the potentate in Heaven. Also I observed that his followers on earth did not seem to get what was promised them here and now. I developed an impish delight in dressing up logical difficulties which my grand aunt could not dispose of. She always slipped back into the logical scheme, and blinked the facts in which I came to take a proprietary interest.

I suppose there is nothing better as a teething-ring for a child who likes logic in the garden variety of Christian theology. I cut my eye-teeth on it with gusto and had not entirely lost interest in that exercise when I went to college.

There I began studying philosophy and economics about the same time. I found no difficulty in grasping the differences between the great philosophical systems as they were presented by our text-books and our teachers. Economic theory was easier still. Indeed, I thought the successive systems of economics were rather crude affairs compared with the subtleties of the metaphysicians. Having run the gamut from Plato to T. H. Green (as undergraduates do) I felt the gamut from Quesnay to Marshall was a minor theme. The technical part of the theory was easy. Give me premises and I could spin speculations by the yard. Also I knew that my “deductions” were futile. It seemed to me that people who took seriously the sort of articles which were then appearing in the Q.J.E. might have a better time if they went in for metaphysics proper.

Meanwhile I was finding something really interesting in philosophy and in economics. John Dewey was giving courses under all sorts of titles and every one of them dealt with the same problem – how we think. I was fascinated by his view of the place which logic holds in human behavior. It explained the economic theorists. The thing to do was to find out how they came to attack certain problems; why they took certain premises as a matter of course; why they did not consider all the permutations and variance of those problems which were logically possible; why their contemporaries thought their conclusions were significant. And, if one wanted to try his own hand at constructive theorizing, Dewey’s notion pointed the way. It is a misconception to suppose that consumers guide their course by ratiocination – they don’t think except under stress. There is no way of deducing from certain principles what they will do, just because their behavior is not itself rational. One has to find out what they do. That is a matter of observation, which the economic theorist had taken all too lightly. Economic theory became a fascinating subject – the orthodox types particularly – when one began to take the mental operations of the theorists as the problem, instead of taking their theories seriously.

Of course Veblen fit fitted perfectly into this set of notions. What drew me to him was his artistic side. I had a weakness for paradoxes – Hell set up by the God of love. But Veblen was a master developing beautiful subtleties, while I was a tyro emphasizing the obvious. He did have such a good time with the theory of the leisure class and then with the preconceptions of economic theory! And the economists reacted with such bewildered soberness! There was a man who really could play with ideas! If one wanted to indulge in the game of spinning theories who could match his skill and humor? But if anything were needed to convince me that the standard procedure of orthodox economics could meet no scientific tests, it was that Veblen got nothing more certain by his dazzling performances with another set of premises. His working conceptions of human nature might be a vast improvement; he might have uncanny insights; but he could do no more than make certain conclusions plausible – like the rest. How important were the factors he dealt with and the factors he scamped was never established.

That was a sort of problem which was beginning to concern me. William Hill set me a course paper on “Wool Growing and the Tariff.” I read a lot of the tariff speeches and got a new side-light on the uses to which economic theory is adapted, and the ease with which it is brushed aside on occasion. Also I wanted to find out what really had happened to wool growers as a result of protection. The obvious thing to do was to collect and analyze the statistical data. If at the end I had demonstrated no clear-cut conclusion, I at least knew how superficial were the notions of the gentlemen who merely debated the tariff issue, whether in Congress or in academic quarters. That was my first “investigation” – I did it in the way which seemed obvious, following up the available materials as far as I could, and reporting what I found to be the “facts.” It’s not easy to see how any student assigned this topic could do much with it in any other way.

A brief introduction to English economic history by A. C. Miller, and unsystematic readings in anthropology instigated by Veblen reinforced the impressions I was getting from other sources. Everything Dewey was saying about how we think, and when we think, made these fresh material significant, and got fresh significance itself. Men had always deluded themselves, it appeared, with strictly logical accounts of the world and their own origin; they had always fabricated theories for their spiritual comfort and practical guidance which ran far beyond the realm of fact without straining their powers of belief. My grand aunt’s theology; Plato and Quesnay; Kant, Ricardo and Karl Marx; Cairnes and Jevons, even Marshall were much of a piece. Each system was tolerably self-consistent – as if that were a test of “truth”! There were realms in which speculation on the basis of assumed premises achieved real wonders; but they were realms in which one began frankly by cutting loose from the phenomena can observe. And the results were enormously useful. But that way of thinking seem to get good results only with reference to the simplest of problems, such as numbers and spatial relations yet men practice this type of thinking with reference to all types of problems which could not be treated readily on a matter-of-fact basis – creation, God, “just” prices in the middle ages, the Wealth of Nations in Adam Smith’s time, the distribution of incomes in Ricardo’s generation, the theory of equilibrium in my own day.

There seem to be one way of making real progress, slow, very slow, but tolerably sure. That was the way of natural science. I really knew nothing of science and had enormous respect for its achievements. Not the Darwinian type of speculation which was then so much in the ascendant – that was another piece of theology. But chemistry and physics. They had been built up not in grand systems like soap bubbles; but by patient processes of observation and testing – always critical testing – of the relations between the working hypotheses and the processes observed. There was plenty of need for rigorous thinking, indeed of thinking more precise than Ricardo achieved; but the place for it was inside the investigation so to speak – the place that mathematics occupied in physics as an indispensable tool. The problems one could really do something with in economics were problems in which speculation could be controlled.

That’s the best account I can give offhand of my predilection for the concrete. Of course it seems to me rather a predilection for problems one can treat with some approach to scientific method. The abstract is to be made use of it every turn, as a handmaiden to help hew the wood and draw the water. I loved romances – particularly William Morris’ tales of lands that never were – and utopias, and economic systems, of which your father’s when I came to know it seemed the most beautiful; but these were objects of art, and I was a work man who wanted to become a scientific worker, who might enjoy the visions which we see in mountain mists but who trusted only what we see in the light of common day.

* * * *

            Besides the spice of rationalizing which doubtless vitiates my recollections – uncontrolled recollections at that – this account worries me by the time it is taking, yours as well as mine. I’ll try to answer the other questions concisely.

Business cycles turned up as a problem in the course of the studies which I began with Laughlin. My first book on the greenbacks dealt only with the years of rapid depreciation and spasmodic war-time reaction. I knew that I had not gotten to the bottom of the problems and wanted to go on. So I compiled that frightful second book as an apparatus for a more thorough analysis. By the time it was finished I had learned to see the problem in a larger way. Veblen’s paper on “Industrial and Pecuniary Employments” had a good deal to do with opening my eyes. Presently I found myself working on the system of prices and its place in modern economic life. Then I got hold of Simmel’s Theorie des Geldes – a fascinating book. But Simmel, no more than Veblen, knew the relative importance of the factors he was working with. My manuscript grew – it lies unpublished to this day. As it grew in size it became more speculative. I was working away from any solid foundation – having a good time, but sliding gaily over abysses I had not explored. One of the most formidable was the recurring readjustments of prices, which economists treated apart from their general theories of value, under the capitation “Crises.” I had to look into the problem. It proved to be susceptible of attack by methods which I thought reliable. The result was the big California monograph. I thought of it as an introduction to economic theory.

* * * *

            This conception is responsible for the chapter on “Modern Economic Organization.” I don’t remember precisely at what stage the need of such a discussion dawned upon me. But I have to do everything a dozen times. Doubtless I wrote parts of that chapter fairly early in other parts late as I found omissions in the light of the chapters on “The Rhythm of Business Activity.” Of course, I put nothing in which did not seem to me strictly pertinent to the understanding of the processes with which the volume dealt. That I did not cover the field very intelligently, even from my own viewpoint, appears from a comparison of the books published in 1913 and 1927. Doubtless before I am done with my current volume, I shall be passing a similar verdict upon the chapter as I left it last year.

* * * *

            As to the relation between my analytic description and “causal” theory I have no clear ideas – though I might develop some at need. To me it seems that I tried to follow through the inter-lacing processes involved in business expansion and contraction by the aid of everything I know, checking my speculations just as far as I can buy the data of observation. Among the things I “know” are the way in which economic activity is organized in business enterprises, and the way these enterprises are conducted for money profits. But that is not a simple matter which enables me to deduce certain results – or rather, to deduce results with certainty. There is much in the workings of business technique which I should never think of if I were not always turning back to observation. And I should not trust even my reasoning about what business men will do if I could not check it up. Some unverifiable suggestions do emerge; but I hope it is always clear that they are unverified. Very likely what I try to do is merely carrying out the requirements of John Stuart Mill’s “complete method.” But there is a great deal more passing back and forth between hypotheses and observation, each modifying and enriching the other, than I seem to remember in Mill’s version. Perhaps I do him injustice as a logician through default of memory; but I don’t think I do classical economics injustice when I say that it erred sadly in trying to think out a deductive scheme and then talked of verifying that. Until science has gotten to the stage of elaborating the details of an established body of theory – say finding a planet from the aberrations of orbits, or filling a gap in the table of elements – it is rash to suppose one can get an hypothesis which stands much chance of holding good except from a process of attempted verification, modification, fresh observation, and so on. (Of course, there is a good deal of commerce between most economic theorizing and personal observation of an irregular sort – that is what has given our theories their considerable measure of significance. But I must not go off into that issue.)

* * * *

           […] when writing the first book about business cycles I seem to have had no clear ideas about secular trends. The term does not appear to occur in the index. Seasonal variations appear to be mentioned only in connection with interest rates. Of course certain rough notions along these lines may be inferred; but not such definite ideas as would safeguard me against the errors you point out. What makes matters worse for me, I was behind the times in this respect. J. P. Norton’s Statistical Studies in the New York Money Market had come out in 1902. I ought to have known and make use made use of his work.

That is only one of several serious blemishes upon the statistical work in my 1913 volume. After Hourwich left Chicago, and that was before I got deep into economics, no courses were given on statistics in my time. I was blissfully ignorant of everything except the simplest devices. To this day I have remained an awkward amateur, always ready to invent some crude scheme for looking into anything I want to know about, and quite likely to be betrayed by my own apparatus. I shall die in the same sad state.

 

[…]

Ever yours,
Wesley C. Mitchell.
(Copy by J.M.C.)

 

Source: Columbia University Archives. Wesley Clair Mitchell Collection, Box 8 “Ch-Ec”, Folder “Clark, John Maurice: v.p., 8 Apr 1926 & 21 Apr 1927. To Wesley C. Mitchell 2 a.l.s. (with related material)”.

Image Source: Foundation for the Study of Cycles.